Download as pdf or txt
Download as pdf or txt
You are on page 1of 71

Saber Revision for IM Prometric v.

1
b. Antihistamine
‫ أنب جمعث األسئلة بس وعلقث‬: ‫جنىيه هبم‬ c. Change the antidepressant to SSRI
‫على بعضهب و رجبءا راجعىا اإلجبببت كىيس‬ d. Thiazide diuretics
e. Audiometry

CARDIOLOGY 6.High
except:
output HF causes includes all

1.Patient come within 3 hrs C/O Lt sided a. Anemia


b. Mitral Regurgitation (MR)
weakness, examination revealed Lt side
c. AV fistula
hemiparesis, pulse 120/min irregular d. Paget’s disease
Oxford - clinical medicine (OHCM 9e) p 128
with diastolic murmur at mitral area.
1st step of management:
a. Heparin
b. Digoxin
c. EEG
d. Carotid angiography 7.Premature ventricular contraction
e. Echo (PVC),all are true except:
a. If anti-arrhythmic given after MI for protection
2.Pt had arthritis in two large joint & of PVC --> decrease chance of sudden death.
b. Use of anti-arrhythmic type I increase mortality.
pansystolic murmur ( carditis ) ,Hx of
c. PVC in normal population doesn’t increase risk
URTI the most important next step: of sudden death.
a. ESR
b. ASO titre 8.Premature ventricular contracture
c. Blood culture
(PVC), all are true except:
a. Use antiarrhythmic post MI improve prognosis
3.The following are features of rheumatic
b. Use of anti-arrhythmic type 1 increase mortality
fever, except: c. Antiarrhythmic drugs should be avoided in
a. Restless, involuntary abnormal movements. patients with asymptomatic ventricular
b. Subcutaneous nodules. arrhythmias after MI, since mortality risk
c. Rashes over trunk and extremities. increases.
d. Short PR interval on ECG. MTB p 74
e. Migratory arthritis

4.years old lady on …….., feels dizzy on


standing, resolves after 10-15 minutes
on sitting, decrease on standing, most
likely she is having : 9.MI with premature ventricular
a. orthostatic hypotension
contractions, the best Tx:
a. Digoxin.
5.what is the most appropriate treatment
b. Lidocaine
for the above patient : c. Quinidine.
a. Antiemetic

SABER internal medicine prometric | P a g e 1


‫ بس لو ىتدي حاجة يبقى ليدوكايين ولو فشل‬، ‫السؤال ده ضد اللى قبلو‬ MTB p31
.‫إدي أميودارون‬

10. A patients ECG showed anterior wall Ml


with PVCs he’s on digoxin, warfarin ,
Lasix what treatment would you like to
add:

11. Patient had anterior wall MI and will


he was transferred to ICU the nurse 15. A 61 years old man with known
notice that he has PVC @ 20 per minute. ischemic heart disease and peripheral
He is on digoxin, diuretic. What do you vascular disease is started on an ACE
want to add? inhibitor by his GP for hypertension. Three
a. Propranolol
weeks later he is admitted with increasing
b. Amiodarone
c. Amoxillin confusion and vomiting. Investigations
d. Nothing reveal: CBC Hb 14.9 g/dI, MCV 88 fI,
WBC 13.6 x 109/L; U & Es: Na+ 131
12. A 60 year old male presented with Hx
mmol/L, K+ 7.3 moI/L, urea 37.8
of 2hrs chest pain ECG showed ST
mmol/L, Cr 858 umol/L. The patient is
elevation on V1-V4 with multiple PVC &
suffering from:
ventricular tachycardia. The management
a. Diabetic nephropathy
is: b. Pheochromocytoma
a. Digoxin c. Polycystic kidney disease
b. Lidocaine d. Raised intracranial pressure
c. Plavix & morphine e. Renal artery stenosis
d. Amiodarone
16. Which of the following is the least likely
13. A 48 years old female with long
to cause infective endocarditis:
standing infection present with a. ASD
bradycardia, your management will be: b. VSD
a. IV fluids c. Tetralogy of Fallot
b. Atropine d. PDA
c. Dopamine OHCM 9e p144
*Bradycardia ttt is the first priority

14. Indication for valve replacement in


infective endocarditis include all except:
a. Viral endocarditis
b. Resistant bacterial endocarditis
c. Fungal endocarditis
d. Aortic valve regurgitation
SABER revision for IM prometric exam| P a g e 2
17. Which of the following is believed to 20. Coarctation of the aorta is commonly
reduce mortality in CHF? associated with which of the following
a. Digitalis syndromes?
b. Furosemide
a. Turner
c. Enalapril
b. Down
d. Procainamide
c. Edward
e. Aspirin
d. Patau
MTB 2e p81
e. Holtorain
Coarctation of the aorta is the most common cardiac
defect associated with Turner syndrome.

21. A patient came to you & you found his


BP to be 160/100, he isn’t on any
medication yet. Lab investigations showed:
Creatinine (normal) ,Na 145 (135-145)
,K 3.2 (3.5-5.1) ,HCO3 30 (22-30)
What is the diagnosis?
a. Essential hypertension
b. Pheochromocytoma
c. Addison’s disease
For diastolic dysfunction
d. Primary hyperaldosteronism

22. 35 years old woman presented with


exertional dyspnea. Precordial
examination revealed loud S1 and
rumbling mid-diastolic murmur at apex.

18. All may cause increased BP except: Possible complications of this condition
a. Obesity can be all the following except:
b. High alcohol intake a. Atrial fibrillation
c. Smoking b. Systemic embolization
d. Gout c. Left ventricular failure
e. NSAID use d. Pulmonary edema
e. Pulmonary hypertension
19. All can cause secondary hyperlipidemia ‫السؤال ده مش مظبوط ألن كل االختيارات ممكن تحصل‬
OHCM 9e p44
except:
a. Hypothyroidism
b. Alcoholism
c. Nephrotic syndrome
d. Estrogen therapy
e. Hypertension
OHCM 9e p 704

SABER internal medicine prometric | P a g e 3


a. If 2/3 of cuff -* false high BP
b. Internal cuff must cover 80% of arm
c. Follow circadian vary 4 late night high BP
d. High BP—* 3 standard deviation away from
normal
e. You have to have more than one reading to Dx
high BP

26. All of the following causes secondary


23. A 70 years old male was brought to the
HTN, except:
emergency with sudden onset of pain in a. Pheochromocytoma
his left lower limb. The pain was severe b. Addison’s disease
c. Hyperaldosteronism (Conn’s disease)
with numbness. He had an acute
d. Renal disease
myocardial infarction 2 weeks previously e. Pregnancy
f. Primary hypothyroidism
and was discharged 24 hours prior to his
MTB p 345,346
presentation. The left leg was cold and
pale, right leg was normal. The most likely
diagnosis is:
a. Acute arterial thrombosis
b. Acute arterial embolus
c. Deep vein thrombosis
d. Ruptured disc at L4-5 with radiating pain
e. Dissecting thoraco-abdominal aneurysm

24. 70 years old woman has had Ml. 2 days


27. Most important complication of
after admission she developed abdominal
systemic hypertension:
pain and diarrhea with passage of blood.
a. Pontine infarction
Abdomen x-ray showed distended b. Renal artery stenosis
intestine with no fluid level. Serum c. Subdural hemorrhage
d. Intracerebral hemorrhage
amylase level slightly elevated with mild
fever. The diagnosis is: 28. Complications of systemic hypertension
a. Ulcerative colitis
are all except:
b. Acute pancereatitis
c. lschemic colitis a. Intracerebellar hemorrhage
d. Diverticulitis b. Renal artery stenosis (this causes HTN)
e. Phenindione-induced colitis (*phenindione is an
anticoagulant) 29. S3 can be auscultated in all, except:
Phenindione [an anticoagulant chemically related to a. Mitral stenosis
anisindione that is no longer available] b. Mitral regurgitation
c. Lt sided heart failure
25. Blood pressure, all of the following are d. Thyrotoxicosis

true except:

SABER revision for IM prometric exam| P a g e 4


30. Patient with chronic atrial fibrillation 36. In atrial fibrillation and stroke, all are
more than 6 months, all can be given true , except:
except: a. Aspirin can be given in AF for prevention of
stroke.
a. Cardioversion
b. Warfarin can be given in AF for prevention of
b. Heparin
stroke.
c. Digoxin
c. Non valvular AF can cause stroke.

31. Treatment of unstable angina includes


37. The following murmur can be
all except:
accentuated by postioning of the patient:
a. Heparin
a. Aortic regurgitation by sitting
b. Nitroglycerin
b. Venous hum by lying down
c. β-blocker
c. Pericardial rub by sitting
d. Aspirin
d. Outflow innocent murmur by sitting
e. ……..

38. ECG stress test is indicated in the


32. The first symptom of left heart failure is:
a. Orthopnea following except:
b. Edema a. Routine (yearly) test in asymptomatic patients
c. Dyspnea on exertion b. In high risk jobs
c. 40 year old patient before starting exercise
33. The first symptom in patient with Lt program

heart failure:
39. Pulmonary stenosis:
a. Orthopnea
a. Commonest lesion in Turner ’s syndrome
b. PND
b. P2 is loud indicates severe stenosis
c. Pedal edema
c. Ejection click means it is valvular
d. Exertional dyspnea. ??
d. All of the above

34. The first symptoms of LHF is


40. Positions that increased or decreased
a. Orthopnea
b. Dyspnea on exertion murmur of Rt & Lt valvular lesions of the
c. Oedema heart

35. Patient with low grade fever and


41. HOCM gradient & murmur factors
arthralgia for 5 days, presented with
increase& decrease it
pansystolic murmur at the apex. H/O
difficulty in swallowing with fever 3 wks
back. Most likely diagnosis:
a. Bacterial endocarditis
b. Viral myocarditis
c. Acute rheumatic fever
d. Pericarditis

SABER internal medicine prometric | P a g e 5


42. ECG inferior MI MS = calcified mitral valve
MVP = mid-systolic click ± late systolic murmur
Corpulmonale = MR
43. MI + HF what has no benefit
a. Diuretics
b. Ace inhibitor 50. Patient has chest pain at rest and
c. Beta blockers sometimes ST elevation management?
d. CCB a. Aspirin
44. DM + HTN Rx b. Nitroglycerine
a. Losartan c. coronary intervention

45. A case of pericardial effusion which is 51. All true about HOCM except:
true Sudden death is rare
a. In Dressler syndrome no response to NSAID
52. Prophylaxis of IE given to
46. CT showing pericardial calcification + a. Mild AR
bilateral pleural effusion+ features of b. MS
c. Valvulopathy after surgery
tamponad, Rx?
MTB 2e p32
a. Steroid
b. Surgery

47. Palpitation with stress O/E systolic click


how to DX
a. ECG
b. ECHO
c. Electrophysiology

48. Valvular lesion that is most occurs as


congenital lesion
a. MS
b. PS 53. After 6h of admission because of recent
c. AS
d. ASD acute MI the patient has SOB and chest
pain (he gave me X-ray pic with enlarged
49. 20 years old male c/o dyspnea and globular heart otherwise normal) what is
orthopnea , no chest pain no fever , mid- the best management?
diastolic murmur at apex no opening snap a. Diuretic
or click and there is ejection systolic at b. Anticoagulant
c. Pericardial tap
pulmonary area with rales at base of both d. Pericardiectomy
lungs
a. Calcific mitral valve 54. Patient with acute manifestations of
b. MVP left then right sided heart failure ,best ttt:
c. Corpulmonale
a. Diuretics only

SABER revision for IM prometric exam| P a g e 6


b. Salt restriction & diuretics Tako-Tsubo cardiomyopathy (AKA: broken heart
c. Surgery syndrome):
= Acute myocardial damage, most often occurring in
55. WHICH VALVE LESION IS MOSTLY post-menopausal women immediately following an
overwhelming emotionally stressful event →
CONGENITAL?
ballooning and left ventricular dyskinesis.
a. Pulmonary Stenosis Management: beta blockers, ACEIs, no coronary
b. Aortic stenosis occlusion, so no benefit from revascularization.
c. Mitral stenosis

60. HOCM
56. BEST PREDICTOR OF LEFT
a. Nitrates may help in management
VENTRICULAR FAILURE b. B BLOCKERS improve exercise tolerance
a. S3 c. Clenching fist increases murmur
b. Pulses alternans
61. HTN classification
57. Staph epidermis a. 118/82 = Pre-HTN
a. Common in prosthetic valve
b. Frunculosis http://emedicine.medscape.com/article/2172178-
c. Cellulitis overview

58. Enterococcal endocarditis, what ttt:


a. Penicillin G
b. Penicillin G + Gentamicin
c. Ceftriaxone
d. Ampicillin + Gentamicin
OHCM 9e p145

62. A case of young male Attacks of


palpitations dizziness

63. Clinically Mitral valve prolapse BEST.


DIAGNOSED by:
a. ECHO
b. EPS
c. ECG
d. STRESS ECG
59. Takutsubo cardiomyopathy:
64. BEST DETERMINANT OF BLOOD
a. May cause coronary obstruction
b. Cause ST Elevation in anterior leads PRESSURE
c. Cause irreversible left ventricular enlargement a. Cardiac Output
b. Peripheral Resistance

SABER internal medicine prometric | P a g e 7


65. AN ECG 69. What is the most common site of
a. LVH. by voltage criteria atherosclerosis
a. Internal carotid artery
66. Patient has mid-systolic click and late b. Vertebral artery
systolic murmur that decreased with c. Aortic and iliac
d. Femoral artery
squatting WHAT IS THIS e. None of above
a. MVP
b. MR 70. what find in macula in infective
c. Restrictive cardiomyopathy
d. AS endocarditis
e. Lt atrial myxoma a. Roth spots
MTB p 96 b. Janewery nodule
a. Osler nodes
b. Splinter he
MTB 2e p30

MTB p 97

71. Asymptomatic patient come for


67. Patient has SOB and Kussmaul’s sign checkup and find pansystolic murmur on
and increased JVP with rapid X and Y apex and echo show EF 60% and mitral
descend YOUR DIAGNOSIS IS: regurge and pulmonary artery wedge
a. Constrictive pericarditis pressure 60 mmHg What to do :
b. LVH
a. Digoxin and diuretic
c. RVF
b. Digoxin and diuretic and ACE I
d. LVF
c. No treatment
e. Myocarditis
d. Valve replace

68. Risk of dissecting aorta


72. According to …………………. BP
a. Pregnancy
b. Marfan syndrome classifications what true
c. Takayasu a. 122/78 normal
d. 2+3 b. 159l95 stage 2
e. All of them c. 128/82 pre-HTN

SABER revision for IM prometric exam| P a g e 8


73. A case of DM and HTN with chest pain b. Aortic regurge
c. Resistant CHF
and ECG demonstrated (raised ST in leads d. Vegatations due to staf epidermide
V2 till V5)
a. Anterior MI
b. lateral MI
c. inferior MI
74. A case of MI with HR 45 (without
ECG) what is the diagnosis
a. Inferior MI
b. Lateral MI
c. Anterior MI

75. A case of DM and HTN ,the best


antihypertensive: 80. The percent of rheumatic cardiac
a. Amlodipine reactivation in rheumatic fever
b. Losartan a. 25%
c. Metoprolol b. 50%
c. 40%
76. Septic shock may be associated with d. 20%
a. increased cardiac output
81. 45 years old patient presented with
77. Patient gains the maximum benefit shortness of breath on exertion for 2
regarding reduction of the LVH form valve years. Headache, dizziness + off and on
surgery of: chest pain. is also present with this ECG
a. Tricuspid
as under:
b. MR
c. AR (ECG shows tall QRS wave in lead V5 and V6 with mild
d. AS ST depression, rest of ECG is normal) . What is the
diagnosis?
NB: I am not sure but I choose AR because it is a. Left ventricular hypertrophy
associated with the most degree of LVH and b. Left axis deviation
diltation so the improvement may be more c. Right ventricular hypertrophy
evident d. Coarctation of the aorta

78. Indications of prophylaxis against 82. Coronary angiography will be carried


endocarditis in dental procedures out in all of the following ,except:
a. Sever aortic stenosis a. Post MI angina
b. Moderate mitral stenosis b. Age > 80 years
c. Valve disease after replacement c. Sever ischemia in stress test
d. All of the above d. Deteriorating LV function

79. Indications of valve replacement in all 83. 60 years old man present with chest
except pain for 6 hours. Previously, he had
a. Endocarditis resistant to antibiotics available
SABER internal medicine prometric | P a g e 9
angina on exertion. The best treatment 90. YOUNG LADY. USED. DRUG TO LOSE
for this patient will be: WEIGHT
a. Nitrates
b. Aspirin 91. PULMONARY HYPERTENSION
c. Percutaneous coronary intervention
d. Beta blockers (sibutramine or fenfluramine)
84. 43 y patient c/o sever chest pain,
92. OLD MALE SHOCKED
attached ECG reveled st segment elevation
a. Cardiac output increased
from V1-V5 Diagnosis? b. Peripheral resistance decreased
a. Ant wall infraction
b. Post wall infraction 93. The cause of death of a case of diffuse
c. Inferior wall infraction
d. Rt ventricular infraction sclerosis with HTN
a. Pulmonary hypertension
85. Treatment of rheumatic fever b. Intracerebral He
c. ARF
a. Penicillin
d. CV stroke
b. Penicillin + steroid
c. Supportive
94. A case has dyspnea with less than
86. 68 ys pat HTN ,AF non rheumatic ordinary effort ,what is the classification
mitral valve how to manage regarding according to the NY classification:
anticoagulant a. I
b. II
a. No need
c. III
b. Aspirin only
d. IV
c. Oral anticoagulant
NYHA
Symptoms
87. Case of AF undergo Cardioversion to Class
Cardiac disease, but no symptoms and no
restore sinus rhythm thers risk for limitation in ordinary physical activity, e.g. no
I
a. systemic embolization shortness of breath when walking, climbing
b. Cerebral embolization stairs etc.
Mild symptoms (mild shortness of breath
c. Peripheral embolization
II and/or angina) and slight limitation during
d. All of above
ordinary activity.
Marked limitation in activity due to symptoms,
88. MI + HF what has no benefit III
even during less-than-ordinary activity, e.g.
a. Diuretics walking short distances (20–100 m).
Comfortable only at rest.
b. Ace inhibitor
Severe limitations. Experiences symptoms
c. Beta b IV
even while at rest. Mostly bedbound patients.
d. CCB

95. 45 years old patient presented with


89. BEST DETERMINANT OF BLOOD
shortness of breath on exertion for 2
PRESSURE
a. Cardiac Output years. Headache, dizziness + off and on
b. Peripheral Resistance chest pain. is also present with this ECG

SABER revision for IM prometric exam| P a g e 10


as under: (ECG shows tall QRS wave in 101. Valve lesion with massive left
lead V5 and V6 with mild ST depression, ventricular hypertrophy
rest of ECG is normal) . What is the a. MR
b. ASD
diagnosis? c. AR*
a. Left ventricular hypertrophy d. Severe HTN
b. Left axis deviation
c. Right ventricular hypertrophy 102. High risk congenital heart disease with
d. Coarctation of the aorta
a. Deteriorating LV function all except:
a. Alcoholic
96. Causes of AR all except: b. Smoking
c. Diabetic
a. Marfan syndrome
d. Congenital
b. Ehler-Danlos syndrome
c. Rheumatic fever
d. Lutembacher syndrome * 103. Case of IE, organism? Moderate
resistance to penicillin, ttt?
97. Verapamil, which type of anti a. Vancomycin + gentamycin
arrhythmics? b. Penicillin + gentamycin
c. Penicillin + vancomycin
a. Type I
b. Type II
c. Type IV * 104. Drugs increasing preload and
d. Type III afterload all except:
e. Non
a. ACEIs
b. Prazocin*
98. Drugs increasing Q-T all except c. Nitrates
a. Amirone
b. Quinidine 105. Patient with chest pain post MI all
c. Procainamide
d. Dispyradole * except?
a. Dressler’s syndrome*
99. Orthostatic hypotension: b. Post infarction angina
c. Acute pericarditis
a. Indicate early renal impairment
d. Embolus
b. Indicate Minimal change GN
c. Females > Males
d. Associated with gastroparesis in DM* 106. Patient with MI, 3 days later: shock,
new systolic murmur, diagnosis?
100. The least investigation needed in acute a. Papillary ms rupture
MI: b. Rupture of free ventricular wall*
c. Papillary ms dysfunction
a. Thallium 21
d. Pericarditis
b. CKMB
c. ECG
d. Pyrophosphate*

SABER internal medicine prometric | P a g e 11


107. Male with bilateral LL edema, 8 -ve
hepatojagular reflex?
a. Rt sided HF
b. Hypoalbuminemia
c. RF

108. Least important investigation for MI


a. LDH ??

109. ECG sowing depressed ST in I, V4,


V5, V6, patient with chest pain SOB, and
headache
a. LVH
b. RVH
c. Coarctation of the aorta

110. GE, dehydration, confusion came to


ER, resident inserted CVP for fluid
resuscitation then after few hrs patient
showed dyspnea + confusion, the cause?
a. Pneumothorax
b. Pleural effusion
c.
d.

SABER revision for IM prometric exam| P a g e 12


CHEST
b. Volume of gas that can be expelled in the 1st
second.
c. Volume of gas that can be expelled after
maximal inspiration.
d. Maximal air flow rate in FVC.
e. Maximal air flow in 1 second.
111. A 58 yrs. old male pt. came with HX
115. Asthma after 40 years old. What is
of fever, cough with purulent foul smelling
true?
sputum and CXR showed : fluid filled Could be psychological.
cavity … the most likely diagnosis is : Eosinophiles are increased significantly.
a. Abscess Peak expiratory value change from night to day.
b. TB Oral steroid change the peak expiratory value
c. Bronchiectasis significantly.

112. One of the major factors causing 116. Air Bronchogram is characteristic

physiological hypoxemia is: feature of:


a. Ventilation-perfusion mismatch. a. Pulmonary edema.
b. decreased diffusional capacity of the alveolar b. HMD= Hyaline membrane disease.
membrane. c. Lobar Pneumonia.
c. increase in the level of 2,3 DPG. d. Lung Granuloma.

113. 55 year old male presented to your 117. ln moderate to severe asthmatic

office for assessment of chronic cough. He patient you may find all except:
a. Decrease Po2 <60
stated that he has been coughing for the
b. PCO2 >60
last 10 years but the cough is becoming c. HCO3 decreased
d. IV hydrocortisone relief after few hours
more bothersome lately. Cough productive
e. Dehydration
of mucoid sputum, occasionally becomes
purulent. Past history: 35 years history 118. The most specific investigation for
smoking 2 packs per day. On examination: pulmonary embolism is:
124 kg, wheezes while talking. a. Perfusion scan
b. X-ray chest
Auscultation: wheezes all over the lungs. c. Ventilation scan
The most likely diagnosis is: d. Pulmonary angiography
a. Smoker’s cough
b. Bronchiectasis 119. A 30 year old male presented with
c. Emphysema Hx of left sided chest pain & shortness of
d. Chronic bronchitis
e. Fibrosing alveolitis breath BP 80/50. On examination left
sided chest hyper-resonance. The most
114. Forced vital capacity: likely diagnosis is:
a. Volume of gas that can be expelled after a. Pneumonia with pleural effusion.
inspiration in one minute. b. MI.

SABER internal medicine prometric | P a g e 13


c. Spontaneous pneumothorax exercising. These symptoms are becoming
worse. Which of the following is the
120. History of recurrent pneumonia, foul
prophylactic agent of choice for the
smelling sputum with blood, clubbing:
Bronchiactasis treatment of asthma in these
circumstances?
121. Patient in ER: dyspnea, Rt sided chest a. Inhaled β2 agonists
pain , engorged neck veins and weak b. Oral aminophylline
c. Inhaled anticholinergics
heart sounds , absent air entry over Rt d. Oral antihistamines
lung Plan of Treatment for this patient: e. Oral corlicosteroids
a. IVF, Pain killer, O2
b. Aspiration of Pericardium 125. 25 year old man had fixation of
c. Respiratory Stimulus fractured right femur. two days later he
d. Intubation
e. Immediate needle aspiration chest tube became dyspnic, chest pain and
hemoptysis. ABG: pH 7.5 po2: 65 pCo2:
122. Which of the following radiological
25 initial treatment is:
features is a characteristic of miliary a. Furosemide
tuberculosis: b. Hydrocortisone
a. Sparing of the lung apices c. Bronchoscopy
b. Pleural effusion d. Heparin
c. Septal lines e. Warfarin
d. Absence of glandular enlargement
e. Presence of a small cavity 126. Which one shifts oxyheamoglobin
dissociation curve to the left :
123. Which one of the following regimens is a. Hypoxia.
the recommended initial treatment for b. Acidosis.
c. High altitude.
most adults with active tuberculosis? d. None of the above.
a. A two-drug regimen consisting of isoniazid (INH)
and rifampin 127. All of the following are true about
b. A three-drug regimen consisting of isoniazid,
rifampin, and ethambutol (MyambutoI). pulmonary embolism, except:
c. A four-drug regimen consisting of isoniazid, a. Normal ABG.
rifampin, pyrazinamide and ethambutol. b. Sinus tachycardia is the most common ECG
d. No treatment for most patients until infection is finding.
confirmed by culture. c. Low plasma D-dimer is highly predictive for
e. A five-drug regimen consisting of Isoniazid, excluding PE.
rifampin, pyrazinamide, ethambutol and d. Spiral CT is the investigation of choice for
ciprofloxacin diagnosis.
e. Heparin should be given to all pts with high
124. A 24 year old woman develops clinical suspicion of PE.

wheezing and shortness of breath when


128. In mycoplasma pneumonia, there
she is exposed to cold air or when she is
will be:

SABER revision for IM prometric exam| P a g e 14


a. A (+ve) cold agglutinin titre. a. Insidious onset exertional dyspnea.
b. Lobar consolidation. b. Bibasilar inspiratory crepitations in physical
examination.
129. The treatment of community acquired c. Haemoptysis is an early sign.
d. Total lung volume is reduced.
pneumonia is:
a. First generation cephalosporin. 134. Primary TB:
b. Penicillin G + second generation cephalosporin.
a. Usually involves upper lobe of lung.
c. Erythromycin.
b. Normal X-ray.
d. Erythromycin + Gentamycin.
c. +ve PPD test.
d. None of the above.
Only occasionally do all the pieces of the diagnostic
e. All of the above.
puzzle fit together easily to yield an etiology of
community acquired pneumonia CAP; hence, therapy
is empiric most of the time. For empiric outpatient 135. Transfer factor for CO (TLCO) is
therapy, a macrolide alone is appropriate. reduced in all of the following conditions
except:
130. All indicate severity of bronchial
a. Sarcoidosis
asthma , except: b. Systemic sclerosis
Intercostal and supraclavicular retraction c. Asthma
Exhaustion d. Berylliosis
PO2 <6OmmHg
P02< 60 mmHg +PCO2> 45 mmHg 136. 20 years old male developed fever for
Pulsus paradoxis > 2OmmHg
2 weeks. Cough + SOB is also present for
131. a 25 year old pregnant presented the same duration. On examination ↓
with fever and sore throat (in flu season) vocal fremitus in the lower chest,
then she developed nonproductive cough percussion note is dull in the same region.
and dyspnea, she was extremely hypoxic, What is your diagnosis?
the most likely Dx: a. Consolidation
b. Collapse of the lower lobe
a. Staph. Pneumonia
c. Pleural effusion
b. Strep. Pharyngitis
d. Pericardial effusion
c. Pneumococcal pneumonia
d. Viral pneumonia
137. A young woman is diagnosed as
132. interstitial lung disease all of the patient of bronchial asthma and on
following are true except: albuterol inhaler as per requested basis.
a. Insidious onset exertional dyspnea She has not developed any acute
b. Bibasilar inspiratory crepitation in physical
examination exacerbation during past 2 years. She has
c. Haemoptysis is an early symptom no significant night time symptoms but
d. Total lung capacity volume is reduced
she has to take albuterol inhaler 3

133. Interstitial lung disease All true times/day. What will be the next step in

except: her management?


a. Regular addition of oral albuterol
SABER internal medicine prometric | P a g e 15
b. ↓ dose of albuterol inhaler b. Respond to penicillin
c. Low dose steroid inhaler
d. Steroid +long term B2 agonist inhaler 146. IPF (Idiopathic Pulmonary fibrosis)
best investigation?
138. ABPA diagnosis then Rx
a. CXR
b. High resolution CT
139. COPD most effective measure = c. MRI
smoking cessation
147. Primary spontaneous pneumothorax
140. Asthma well controlled on high dose common cause
inhaled steroid + LABA what to do a. Rupture of Bulla
b. Asthma
a. Stop steroid
c. COPD
b. Stop LABA
c. Reduce steroid
If you are asked about primary = bulla
d. Continue same
If you are asked about secondary = COPD
If you are asked about most common cause = COPD
141. Asthmatic Rx with beclomethasone
inhaler then response measured by (FeNo 148. Malignant effusion done aspiration &
test ), FeNO 55ppp normal <25 which pleurodesis, now recurrent effusion, what
value indicate most significant to do:
improvement a. Aspiration
a. 5 b. Fibrinolytic
b. 10 c. Pleurectomy
c. 15
d. 20 149. the most common cause of
FeNO = Fractional excretion of NO spontaneous pneumothorax
a. Bronchial asthma
142. 70 yr smoker since 20 yr with grade b. COPD
4 clubbing c. Bronchiactasis
a. COPD d. Ruptured bulla
b. Bronchogenic CA
150. Pulmonary TB suspected , what
143. TB DRUG RX? confirms:
a. CXR
144. 14 yrs. boy chronic dry cough b. CT chest
c. Sputum acid fast bacilli
examination & CXR normal which is true d. Tuberculin test
a. DLCO 50%
b. Diurnal variation in PEFR 10%
151. CASE OF PULMONARY
c. PO2 9.4 kpa
TUBERCULOSIS WIH RENAL
145. in atypical pneumonia what is true IMPAIRMENT
a. Usually no correlation between clinical finding a. No streptomycin
and CXR

SABER revision for IM prometric exam| P a g e 16


152. patient with dyspnea cough, bilateral b. Thoracocentesis

crackles , bilateral infiltrates on CXR,


159. SARCOIDOSIS CASE best diagnostic
diagnosis:
(no ACE among options)
a. ARDS
a. Transbronchial biopsy
b. Pneumonia
c. Asthma
d. COPD 160. BRONCHIECTASIS UNLIKELY
ORGANISM
153. Farmer presented with mild dyspnea a. Anaerobes
cough, mild crackles: b. Staph
c. Strept
a. Allergic pneumonitis
d. Pseudomonas
b. Idiopathic pulmonary fibrosis
c. TB
161. COMMONEST CAUSE OF CANCER
154. Curve of O2 dissociation deviated to LUNG
right, what causes: a. Smoking
b. industrial smoke
d. 2,3 diphosphoglycerol increase
c. pollution

155. What is the most common lung tumor


162. As regarding interstitial lung disease
a. Small
a. Extrinsic allergic alveolitis is associated with
b. Adenocarcinoma
clubbing.
c. Squamous
b. Fibrosing alveolitis almost not come before R A
d. Metastatic
c. Could be associated with ANA
d. ASBESTOSIS CAUSE FIBROSING ALVEOLITIS
In US: Adenocarcinoma
In UK: Squamous cell carcinoma
163. PATIENT known smoker his repeated
156. Causes lung cavitation cough productive high volume increased in
a. T.B the morning and associated with clubbing
b. Asthma
c. Radiation ,hemoptysis, scattered crepitations over
d. Pulmonary edema chest WHAT IS YOUR DIAGNOSIS
a. Ch. Bronchitis
157. SPUTUM CYTOLOGY CONFIRMED b. Pneumonia
MALIGNANCY WHILE X RAY. NORMAL c. Bronchiectasis
d. Bronchogenic carcinoma
reassurance e. ABPA
a. Follow up
b. Brushing biopsy 164. what is the common lung cancer with
higher mucous production
158. PNEUMONIA IN YOUNG MALE for 2
a. Small cell
days HR 120 Temp 40 Pleural effusion b. Large cell
After 2 days c. Adenocarcinoma
d. Alveolar carcinoma
a. Continue IV antibiotics

SABER internal medicine prometric | P a g e 17


165. The best management of exercise b. Replace these 2 drugs with doxycycline and
……….. not remember
a. Inhaled salbutamol
c. Continue other 2 drugs for 6 months
b. Inhaled LABA
d. Add 2 drugs ………. Not remember to 4 drugs
c. Montelukast
d. Theophylline
171. Case of mesothelioma the best
166. Obese patient C/O day time sleepiness confirmatory diagnosis by:
,HTN AND NOCTURIA WHAT IS THE a. Closed pleural biopsy
b. Transbronchial pleural biopsy
2ND STEP c. CT
a. Assurance
b. Assess PO2 172. A case of acute severe asthma not
c. Polysommonography
d. Give anti-hypertensive drug responding to B2 agonist
a. Ipratropium bromide
167. Patient on oral prednisone 30 mg and b. Corticosteroid inhaler
c. Long acting B agonist
b2 agonist and curve drawn show
improvement of PEF after 13 days what 173. What is the most common tumor type
to do : in the posterior mediastinum?
a. Decrease steroid a. Vascular
b. Increase steroid b. Teratoma
c. Add long B2 agonist with increasing Steroid c. Neurological
d. The same dose of steroid
N.B: I answered this wrongly and when I
168. Scenario about male patient 15 yrs searched in the net, I found that the most
common tumor in the anterior mediastinum is
smoker come with chronic cough for 2 teratoma and the most common tumor in the
years and hyperinflated chest in x ray posterior mediastinum is the neurological
tumors
what is diagnosis :
a. COPD
174. What can you give with COPD
exacerbation?
169. indication of surgery in spontaneous
a. Salmeterol
pneumothorax b. Ipratropium
a. Single large bullae c. Tiotropium (LAMA)
b. Marfan syndrome d. Oral prednisolone. (just for asthma not for
c. Ehler-Danlos syndrome COPD)
d. Catamenial
175. Patient had hip surgery 1 week then
170. scenario about pat has TB and took sudden onset of SOB ex normal ECG show
anti TB drugs 4 drugs but after 8 weeks tachycardia and inverted T wave v1->v4
develop resistant from rifampicin and a. PE
isoniazid what to do b. Acute MI
c. Bronchopneumonia
a. Continue the same treatment for one year

SABER revision for IM prometric exam| P a g e 18


176. Picture of respiratory function test 181. 20 y patient with history of SOB that
awake at night for 3 time how to
177. Patient asthmatic on b2 agonist establish diagnoses of asthma?
occasionally ,…………….., now more than 3 a. Chest x ray
b. History of patient
attacks what u will give c. CT
d. MRI

178. 20 years old male developed fever for 182. Old pat HTN obese cigarette smoker
2 weeks. Cough +SOB is also present for COPD co of SOB (there’s lab and ABG e
the same duration. On examination ↓ type 2 RF respiratory acidosis )what the
vocal fremitus in the lower chest, best improve long life of the pt
percussion note is dull in the same region. a. Bronchodilator
b. Stop smoking
What is your diagnosis?
c. Steroid
a. Consolidation
b. Collapse of the lower lope
c. Pleural effusion 183. COPD with SOB Diabetic HTN BP
d. Pericardial effusion 120/80 RR30 pulse 110 ABG pH 7.2
pco2 65 po2 40 best management
179. Transfer factor for CO (TLCO) is
a. Elective intubation
reduced in all of the following conditions b. Noninvasive ventilation
c. High O2
,Except:
d. Bronchodilator
a. Sarcoidosis
b. Systemic sclerosis
c. Asthma 184. Case of asthma ttt by short and long
d. Berylliosis B2 with SOB what you add?
a. Anti-cholinergic
180. A young woman is diagnosed as b. Oral steroid
patient of bronchial asthma and on c. Theophylline

albuterol inhaler as per requested basis.


185. Patient with aspirin induced asthma
She has not developed any acute
with osteoarthritis you can give the pat.
exacerbation during past 2 years. She has a. Piroxicam
no significant night time symptoms but b. Brufen
c. Indomethacin
she has to take albuterol inhaler 3
d. Meloxicam
times/day. What will be the next step in ‫االختيارين ينفعوا‬
her management? 186. Case of bronchiectasis, diagnoses?
a. Regular addition of oral albuterol a. X ray chest
b. ↓ dose of albuterol inhaler b. Bronchography
c. Low dose steroid inhaler c. High resolution CT
d. Steroid +long-term B2 agonist inhaler

SABER internal medicine prometric | P a g e 19


187. Case of erythematous skin lesion on
195. O2 dissociation curve to Rt or left?
both chins x ray chest bilateral hailer
lymphadenopathy Dx?
a. Sarcoidosis 196. 15 ys child, received steroids for
b. Histioplasmosis asthma, what to do next?
a. Exhaled NO
188. Asthma well controlled on high dose b. Sputum induction
inhaled steroid + LABA what to do c. Pulmonary function tests
a. Stop steroid
b. Stop LABA 197. Mesothelioma?
c. Reduce steroid
d. Continue same
198. Contraindications for surgery foe
189. 70 yrs smoker since 20 yr with grade cancer lung, LN mediastinal 3 cc
4 clubbing a. Hoarseness of voice, vocal cord affection*
a. COPD
b. Bronchogenic CA 199. Bronchial asthma
a. Residual volume increases*
190. TB DRUG RX? b. Increased FEV1/FVC
c. Decreased PaO2
d. Improved by cough
191. 14 yrs boy chronic dry cough
examination & CXR normal which is true 200. Alpha 1 antitrypsin all except
a. DLCO 50% a. AD
b. Diurnal variation in PEFR 10% I think b. Affect upper lobe of the lung*
c. Po2 9.4 kpa c. Start with childhood
d. Asthma = high DLCO & RF 2 d. Pancolitis is a major manifestation

192. Commonest SE of anti TB 201. Cavitation in lung all except


a. Optic neuritis a. Cystic fibrosis
b. Renal failure b. Pneumoconiosis*
c. Liver affection c. SCC
d. Staph. A
193. Farmer presented by dyspnea that
increase during work 202. Patient with patchy pneumonia,
a. Work pneumonitis treatment?
b. TB a. Penicillin
c. IPF b. Ceftriaxone
c. Ampicillin clavulonic + cefotaxime
194. Dyspnea, fever, tachycardia, CXR:
pleural effusion, what investigations to do 203. Patient with no sputum, normal CXR,
a. Diagnostic thoracoscopy sputum +ve for malignancy, diagnosis by?
b. a. Madiastinoscopy and biopsy
c. b. Bronchoscopy and biopsy*

SABER revision for IM prometric exam| P a g e 20


c. CXR

204. Dyspnea with large PE, stable?


a. Streptokoinase
b. Thrombectomy
c. IV heparin
d. O2 therapy

205. Indications of surgery in


pneumothorax
a. Single large bulla
b. Ehler-Danlos syndrome
c. Catamanal pneumothorax

206. All are causes of hemoptysis except:


a. Chronic bronchitis*

207. Nosocomial pneumonia, treatment?


a. Ceftriaxone + vancomycine + gentamycin*
b. Ceftriaxone + gentamycin
c. Ceftriaxone + amphotricin B + gentamycin

208. Treatment if TB all except:


a. Steroids indicated at some cases
b. Starting by 4 drugs for 2 months
c. Streptomycin is preferred in TB meningitis*

209. Female patient with BA on albuterol


inhaler as per requested basis but
normally it should be used 4-5 times per
day, she has no exacerbation, she came for
follow up, exam: mild wheezes on the
back, what to do next?
a. Increase albuterol inhaler to 4 times per day
b. Oral albuterol
c. Low dose steroid inhaler

SABER internal medicine prometric | P a g e 21


GIT
214. Complications following pancreatitis
may include all of the following except:
Pulmonary atelectasis.
Altered mental status.
Shock.
210. a 28 year old lady presented with Afferent loop syndrome.
Sepsis.
history of increased bowel motion in the
last 8 months.About 3-4 motions/day 215. All are true in black hairy tongue,
.Examination was normal.Stool analysis except:
showed: Cyst,yeast: nil ,Mucus –ve Hydrocortisone can be used.
Culture:no growth, Whats the most Advice pt not brush his tongue.

likely diagnosis? 216. All true about black hairy tongue,


inflammatory bowel disease
irritable bowel disease except:
diverticulitis
Advice patient not to brush his tongue
211. Fecal leukocytes come with all It is caused by bacterial over growth
except: Bad oral hygiene is a potential cause
Antibiotics may be prescribed if refractory
Shigellosis.
Clindamycin induced colitis.
Idiopathic ulcerative colitis.
217. regarding aphthous ulceration in the
212. Which of the following is the most mouth all are true except:
potent known stimulator of gastric acid there is no treatment for acute ulcer
tetracycline suspension helps in healing
secretion: there is immunological role in its development
Pepsinogen. mostly idiopathic in origin
Gastrin.
Acetylcholine B. 218. Diarrhea can occur in all the
Enterogastrone.
Cholecystokinin following, except:
Hypothyroidism
213. In acute pancreatitis the chief Hyperthyroidism

adverse factor is: 219. 30 years old female with chronic


Hypercalcaemia (> 12 mg/dl).
Age above 40 years. hepatitis B infection, her investigations
Hypoxia. as under: HBV-DNA : undetected ALT :
Hyperamylasemia (> 600 units).
501 U\L Anti HBC : present, The best
Gallstones.
treatment for her will be:
Lamivudine
Interferon α
Lamivudine +interferon α

SABER revision for IM prometric exam| P a g e 22


None of the above NITRATE
NADOLOL
220. This is barium swallow of a patient diltiazem

(picture of achalasia barium swallow).


225. Long scenario, bloody diarrhea, best
What should be the next management
ttt:
step? Metronidazole
Laparoscopic myotomy
Pantoprazole therapy 226. Hepatic encephalopathy, SBP
Antibiotics
Longterm omeprazole therapy confirmed , what ttt:
IV cefotaxime
221. An alcoholic middle aged man
227. Hepatic with flappy tremors &
developed epigastric pain sudden in
jaundice had loose motion for 24 hours
onset with vomiting. On examination:
, lab showed acidosis& azotemia &
tenderness on epigastrium. no other
hypokalemia, what cause this
significant findings. Which will be the
deterioration:
best investigation in this patient?
Acidosis
Serum amylase
azotemia
Serum lipase
hypokalemia
ERCP
loose motion
Endoscopic exam

228. About portosystemic hepatic coma,


222. Alcoholic man developed acute onset
which is wrong:
jaundice, on examination: jaundice, no
There is portal hypertension
other stigmata. Serum bilirubin 8mg/dl, Constipation can contribute to this
ALT 400 U/L, AST 320 U/L , what is GI bleeding is one of important precipitant factors
Normal ammonia exclude encephalopathy
the diagnosis in this patient?
Acute infectious hepatitis
229. Lady came with coffee ground
Alcoholic hepatitis
Acute pancreatitis emesis , endoscope revealed 1 cm clean
Liver abscess gastric ulcer not oozing, what to do:
No further management
223. Gingival hyperplasia is present in all, Epinephrine injection
except: Epinephrine and heat cautery
Cyclosporine therapy Heat cautery
Phenytoin therapy
AML 230. Patient did endoscopy through
ALL which epinephrine injected then
coagulated and clippage done to
224. AGEAL VARICES did band ligation
PROPHYLAXIS

SABER internal medicine prometric | P a g e 23


bleeding source, what was found in 236. PATIENT RECEIVED ANTIBIOTICS
endoscope: AND GOT DIARRHEA
1 cm clean gastric ulcer (PSEUDOMEMBRANOUS COLITIS)
2 cm clean duodenal ulcer
5 cm gastric ulcer with flat spot CLOSTRIDIUM DEFICILE
3 cm gastric ulcer with white patch covering Air borne Isolation.
Reverse Isolation.
231. Patient with Crohn's disease came Droplet Isolation.
Contact Isolation.
with jaundice, high bilirubin , -ve anti-
mitochondrial antibody, what expected 237. HBe Ag ALL TRUE EXCEPT
to be found: Absence means. No ch infection
c-ANCA but pANCA
Anti ds DNA 238. HEMATEMESIS due to NSAIDs
Anti thyroperoxidase
Anti liver soluble antigen +pANCA endoscopy :superficial ulcer
Crohns and UV mostly with PSC but Coeliac with epinephrine injection
autoimmune hepatitis clipping of bleeder!!
PPI + tranexamic acid
232. Patient with multiple gall stones, PPI+oral sucralfate

presented with epigastric tenderness,


239. HEMATEMESIS IN A CIRRHOTIC
vomiting , best prevention by:
PAIENT
Cholecystectomy
TIPS
ENDOSCOPY
233. Young student after a party
(alcohol) Fever (40) agitation 240. ALCOHOLIC PAIENT BUT ALT
what is the Treatment? HIGHER
Diazepam Virology markers
check VIRAL markers
234. Alcohol toxicity presentation
includes all except: 241. CARCINOID TUMOR BEST
Hypothermia DIAGNOSED BY
Urinary 5-HIAA levels
scintigraphy for somatostatin receptors
235. HEPATITIS A complications chromogranin

Myocarditis 242. regarding to the shock


Pancreatitis
In septic shock CO increased 2-in cardiogenic shock
Cholecystitis
PCWP increased
Peritonitis
in hypovolemic shock PH >7.3
(Relapse, Cholestasis
13 AND 14 ARE SAME IDEA
Acute liver failure: Fulminant hepatitis A
(c. oedema), it doesn't become chronic)

SABER revision for IM prometric exam| P a g e 24


243. 50 ys old man his father died 249. 27 yrs male with HBV DNA
because of colon cancer THE BEST undetected, liver enzymes normal.
WAY FOR SCREANING IS HBsag +ve ,HBeag +ve . What to do
FOB Lamivudine
Sigmodioscopy Interferon
barrium enema No treatment
barrium meal Lamivudine and interferone
colonscopy
250. Most common cancer with HCV
244. H.PYLORI WHAT IS WRONG NON Hodgkin lymphoma
G –VE flagilate bacilli
can be erredicated in 2 wks of 251. A case of epigastric pain not
bisthmus,meternidazol,tetracycline
maily linning mucosa of the stomach responding to proton pump inhibitors
mainly cause duodenal ulcer and +ve for Giemsa stain:
eradication of it make recurrence is impossible PPI+metronidazole
Amox 1gm+clarithromycin500+PPI20 (TWICE)
245. Ileal resection will decrease level of
which 252. A case of fundoplication after non
VIT B6 responding to PPI counseling with the
VIT B12
FOLIC ACID patient:
IRON Improve after operation to PPI
CALCIUM No benefit

246. most common cause of death in 253. A case of esophageal disease,


acute fulminant hepatisis dysphagia, management:
Acidosis PH measurement
Hepatorenal syndrome CT
Bleeding
Brain oedema 254. Patient is known to have heart
ischemia for which he has underwent
247. HBV curve when patient non
CABG is coming with unrelieved
infectious
6 weeks abdominal pain with no diarrhea or
12 week constipation and with soft slightly
24 week
distended bowel
36 week
Ischemic mesentry
Paralytic ileus
248. Clubbing present in :
Whipple
255. Patient is admitted with COPD and
in the hospital he complained RT
hypochondral pain and positive

SABER internal medicine prometric | P a g e 25


murphys sign and by US there is None of the above
264. This is barium swallow of a patient
thickening of the wall of the gall bladder
(picture of achalasia barium swallow).
with peri-cholecysteal collection
Acute acalculus cholycystitis What should be the next management
Acute calculus cholycystitis step?
Acute emphyseamatus cholycystitis
Laparoscopic myotomy
Pantoprazole therapy
256. Achalasia what is ttt: pneumatic Antibiotics
dilatation Longterm omeprazole therapy

257. Tumors associated with H.pylori 265. An alcoholic middle aged man
developed epigastric pain sudden in
258. Alcoholic patient with cirrhosis AST onset with vomiting. On examination:
300 ALT 400 patient 18 manage, tenderness on epigastrium. No other
bilirubin 6? significant findings. Which will be the
best investigation in this patient?
259. Liver transplantation
Serum amylase
Check hepatic virology
Serum lipase
ERCP
260. All true about IBS except rectal Endoscopic exam
bleeding loss of weight
266. Q about post exposure prophylaxis
261. Patient with ch HBV PCR –ve AST when exposed to blood from pat.
50 HBVC Ab +ve what you give HBSAg+v
no ttt ---antiviral ---- b-interferone HB immunogloben single dose
HB immunogloben in 2 dose and HB vaccine at time
262. Patient with cirrhosis jaundiced us of exposer
showed moderate ascites and Rt pleural HB immunogloben in 2doese and after 30 day
give HB vaccine
effusion ttt?
Chest tube
salt and water restriction 267. Patient with hematochazia PR
reveled blood clot colonscopy bleeding
263. 30 years old female with chronic from diverticulum, Best management
hepatitis B infection, her investigations tattoo the site of bleeding then refer to surgery
as under: HBV-DNA : undetected ALT : inject adrenaline then hemicloctomy
-………………………….( uncomplicated diverticulitis
501 U/L Anti HBC : present The best
respond to conservative therapy with bowel rest.
treatment for her will be: low-fiber diet .. If bacterial infection is
Lamivudine suspected, antibiotics… Surgery is often not needed.
Interferon α Complications, such as peritonitis, abscess,
Lamivudine +interferon α
or fistula may require surgery)

SABER revision for IM prometric exam| P a g e 26


Occult blood
268. Case of malabsorption definitive OGD
Barium
diagnoses made by barium full throw of
small intestine 276. Causes of enlarged pulsatile liver
colonscopy e multiple biopsy Haemochromatosis
Fat content of stool HCC
breath test Alcoholic cirrhosis

Pulsatile liver =
269. pseudomembranous colitis RX
Tricuspid regurgitation
270. pseudomembranous colitis dx Hepatocellular carcinoma
Vascular abnormalities
271. Young male with abd tenderness Aortic transmission

and bloody diarrhea colonscopy ext.


277. 12 female with jaundice & itching
colitis up to sigmoid RX
Anti mitochondrial +ve Rx
Methylpred Steroid
Oral pentasa UDCA
Rectal mesalamine

278. Ulcerative colitis skin lesion all


272. woman with J & anti mitochondrial
except
+ve Rx
Pyoderma lipodica
Ursodeoxycholic acid
Pyoderma gangrenosum
HP
Erythema nodosum
Increased incidence of gastric CA
Erythema multiformis
Rx I week with ppi & 2 antibiotic 90% success
Acquired during childhood
All above 279. Precancerous lesion of colorectal
cancer except:
273. Most part of colon prone to Polyposis
ischemia Diverticulosis*
UC
Transv. colon
Ileal resection
Hepatic flex
Vit. B12 defeciency*
Splenic flex

280. Diagnosis of gastritis one is false


274. known HCV presented with UGIB
Barium meal showing significant changes*
given fluid iv omeprazole and
vasopressin what to do next 281. Chronic HCV lead to
U/S 1 Pancreatic adenocarcinoma
OGD 2 Cancer liver
Observation
TIPSS 282. Celiac dse involves
275. Abdominal bloating + HB of 9 gram Hepatic flexure of the colon
what to do

SABER internal medicine prometric | P a g e 27


283. Colorectal cancer all except: 3
4
Associated with acromegaly
Monosat F acid diet
Polyposis 292. Pruritis
Age > 70 1 Common in DM
Wilson’s disease 2
3
284. Rancon criteria of acute pancreatitis 4

all except:
293. Most accurate tool for HCC
Increased Sr. Amylase*
Alfa fetoprotein
CT
285. Increased LDL, decreased HDL, US
increased TG 400 MRI
Parovastatin
Fibrates 294. Hydatid cyst in the liver
Nicotinic acid Hx of contact with a dog
Omega 3 Using antimalarial chloroquine

286. A case of pigastric pain,


investigations?
Abd. US
Abd.XR erect
AXR lateral decubitus
ECG
All

287. H.pylori treatment?


Clarithromycin + amoxil + PPI*

288. Non palpable purpura all except


Henoch-showenlin purpura *
Osteomalacia

289. EM of SLE renal biopsy


Mesangial
Membranous
Subendothelial
Diffuse GN

290. Polymyositis?

291. Spleen
1 Removal of Howell jelly bodies and Heinz bodies
from RBCs
2
SABER revision for IM prometric exam| P a g e 28
INFECTIONS
295. a 24 yrs old pt. came for check up
after a promiscuous relation 1 month
ago .. he was clinically unremarkable,
VDRL : 1/128 … he was allergic 2
penicillin other line of management is :

296. a 25 yrs old Saudi man presented


with Hx of mild icterus , otherwise ok ..
hepatitis screen : HBsAg +ve , HBeAg
+ve , anti-HBcAg +ve , the diagnosis : 297. What is the least effective AB of the
a- acute hepatitis B
b- convalescent stage of hep. B following to staph. aureus:
c- recovery with seroconversion Hep . B • clindamycin.
d- Hep B carrier • erythromycin.
e- chronic active Hep. B • amoxicillin.
• Vancomycin.
Fewer than 5% of staph. aureus isolates are sensitive
to penicillin.

298. regarding protective measures of


malaria, all true except:
- infestation occur more in day than night :
- using insect repellant is useful

299. Regarding typhoid fever, all are true


except:
• fever and red spots appear on the same time.
• can be completely eradicated even in the presence
of gall stones.
• transmitted by food, milk and water.
• can be treated by quinolones.

300. Within 6 hours after attending a


dinner party, 10 participants developed
sever N/V, abdominal cramp and
diarrhea. Most of them resolved
spontaneously. Few were admitted for

SABER internal medicine prometric | P a g e 29


correction of dehydration. The most diarrhea

probable cause is: Enterotoxigeni


Watery Salads, cheese,
a. Salmonella food poisoning. c Escherichia
diarrhea meats, water
b. Botulism. coli
c. Staphylococcal food poisoning. Ground beef, roast
d. Giardiasis. Enterohemorrh Bloody beef, salami, raw
e. Clostridium perfringes food poisoning. agic E. coli diarrhea milk, raw vegetables,
apple juice
301. A family went to a dinner party Salmonella Inflammato Beef, poultry, eggs,
spp. ry diarrhea dairy products
after 6 hours they all had symptoms of Campylobacter Inflammato
Poultry, raw milk
abdominal pain,nausea, vomiting and jejuni ry diarrhea
Potato or egg salad,
dehydration. Some of them recovered Shigella spp. Dysentery lettuce, raw
while others needed hospitilzation. vegetables
Vibrio
What’s the most likely organism? Mollusks,
parahaemolyti Dysentery
a) giardia crustaceans
cus
b) staph aurens
c) salmonella
d) c . perfiringis 302. The following are characteristics
e) c.boyulism features of brucellosis except:
a. Lymphadenopathy
Bacterial Food Poisoning b. Hepatomegly.
Incubation c. Splenomegaly.
Common Food d. Diarrhea.
Period, Symptoms
Organism Sources e. Backache.
1–6 H
Ham, poultry, 303. in brucellosis, all of the following are
Nausea,
Staphylococcus
vomiting,
potato or egg true except:
aureus salad, mayonnaise,
diarrhea cream pastries a) brucella abortus cause more severe form than B.
Nausea, melitansis in children .
Bacillus cereus vomiting, Fried rice b) human to human is rarely documented .
diarrhea c) human can be infected through inhalation .
d) brucella species are small, non motile gram -ve
8–16 H
coccobacilli .
Abdominal e) pt with high titer can show false -ve .
Clostridium cramps, Beef, poultry,
diarrhea
perfringens (vomiting legumes, gravies 304. A 55 years old male with COPD ,
rare)
Abdominal complaining of 1 wk fever and
cramps, Meats, vegetables, productive cough on CXR showed Left
B. cereus diarrhea dried beans,
(vomiting cereals upper pneumonia, sputum culture +ve
rare) for H.inf, most drug effective is
>16 H a. Pencillin
b. Doxycycline
Vibrio cholerae Watery Shellfish c. Cefuroxime

SABER revision for IM prometric exam| P a g e 30


d. Gentamycin 310. The greatest risk of developing
e. Carbinacillin
chronic hepatitis and cirrhosis occurs
after:
305. Which one of the following diseases a. Hepatitis A infection.
is not transmitted by mosquitoes? b. Hepatitis B infection.
c. Hepatitis C infection.
a)rift valley fever
d. Hepatitis D infection.
b)yellow fever
e. Hepatitis E infection.
c)relapsing fever
d)filariasis
e)dengue fever 311. In a gram-negative bacterial
septicemia:
306. Pt presents with fever swelling is a) pseudomonas is the most common organism
felt,Ant.lymph node swelling warm, involved.
b) Many of the advese changes can be accounted for
tender & fluctuant Dx: by endotoxin.
a) viral infection . c) The cardiac index is low
b) bacterial lymphadenitis . d) Central venous pressure is high.
c) Hodgkin L. e) Endotoxin is mainly a long-chain peptide.
d) ALL .
312. in septic shock:
307. Patient with H/O fever, peripheral a) The mortality rate is 10 to 20%.
blood film +ve for malaria: b) Gram-negative organisms are involved exclusively
c) The majority of patients are elderly
a) Banana shaped erythrocyte is seen in P. vivax
d) The most common source of infection is
b) Mostly due to P. falciparium
alimentary tract.
c) Treated immediately by primaquin 10mg for 3 days
e) Two or more organisms are responsible in the
d) Response to Rx will take 72 hr to appear
majority of cases.

308. The following drugs can be used in


313. HSV type 1 infection of the oral
prophylaxis of malaria in chlorquine-
cavity, all true except:
resistant area, except: a- Is the commonest viral infection in the oral cavity
a) Mefloquin . b- Can give gingivostomatitis
b) Doxycycline . c- In primary infection, there is systemic involvement
c) Proguanil . d- May present with tonsillitis without oral lesion
d) Chlorquine + dapsone + pyrimethamine .
314. A 40 year old white male is
309. Hepatitis most commonly
transferred to your institution in septic
transferred by blood is:
shock less than 24 hours after onset of
a) HBV.
b) HAV. symptoms of a non-specific illness. He
c) HCV (previously known as non a non b). underwent a splenectomy for trauma 5
d) None of the above.
years ago. Antibiotic coverage must be
directed against:
a. Streptococcus, group A.

SABER internal medicine prometric | P a g e 31


b. Klebsiella pneumoniae. 320. complicated malaria jundice +
c. Staphylococcus aureus.
d. Escherichia coli. splenomegaly+anaemia Rx
e. Streptococcus pneumoniae. Oral Chloroquine
IM Chloroquine
315. 30 years old Saudi man referred IV artesunate
Fansidar
from Jazan after tour. He developed
fever and vomiting. On examination he 321. HCV +ve monosexual what
is pale , temp 38 C. Spleen palpable 3 precaution
cm below costal margin. No Condoms
No precaution
hepatomegaly or lymphadenopathy . lab Lamivudine for wife
test shows: WBC : 2.6, platelet: 100, OCP
eosinophil count : 250. What will be the
322. scenario of meningitis G –
diagnosis?
vediplococci Rx
a) Visceral leishmaniasis
b) Enteric fever Ampicillin +Gentamycin
c) Early schistosomiasis Vancomycin
d) Malaria Ceftazidime

316. Chancroid is caused by: 323. women with suprapupic pain &
a) Chlamydia trichomatis burning micturition urine culture
b) Haemophillus ducreyi candida albicans Rx
c) Gardenella vaginalis
Flucanzole
d) Calymatobacterium granulomatis
Itracanzole

317. Drug of choice in acute cystitis is:


324. diabetic c/o headache o/e black
a) Trimethoprim-sulfamethoxazole
b) Ceftriaxone eschar at roof of the mouth had block
c) Ciprofloxacin 500mg 12hourly for 3 days Rt nostril scrapping of black lesion and
d) Nitrofurantoin
microscopy fungal hyphae Rx
318. gamainterferon used for Itracanazole
Iv fluconazole
Mantoux
Iv amphotericin
Latent TB
Active TB
Monitor RX 325. Infectious organism following dogs &
cats bite within 24 hour
319. HIV+ Tongue leukoplakia (picture) Staph aureus
CDC 300 class as Pasterullamultocida
A3
B2 326. PEP for needle stick injury from a
C3 pt. that is HBV +ve
C

SABER revision for IM prometric exam| P a g e 32


327. COPD+blood film G –ve diplococci ? Voriconazole
Fluconazole
Strep
Amphotericin B
Hemophilus
itraconazole
Staph
Fluconazole is still considered a first-line agent in
Moraxella
nonneutropenic patients with candidemia or
suspected invasive candidiasis. treatment of invasive
328. AIDS patient , CD4=12, what candidiasis in clinical trials appears to favor use of
prophylaxis echinocandins
Azithro+doxycyclin
Azithro+ampicillin 334. ALKALINE URINE WHAT IS THE
Azithro+cipro EXPECTED BACTERIAL INFECTION?
Proteus
329. 42 farmer present with 1 week
fever headache malaise, tender cervical 335. 35 ys old male patient c/o
lymphadenopathy, splenomegaly,sore suprapubic pain ,dysuria,urine analysis
throat: revals full field pus WHAT IS THE BEST
Brucellosis MANGMENT
Schistosmiasis
1-one dose of TMP/SMX
Leishmaniasis
2-emperical quinolone for 3 days
3-emperical quenilone for 14 days 4- alkalinzation
330. Patient with fever, cervical of urine
lymphadenopathy, scattered enlarged
lymph nodes hepatosplenomegaly, +ve 336. 20 ys old nurse concerning of risk

monospot test: for HIV transmission after she received


Lymphoma needle stick from known pt of HIV
Infectious mononucleuosis WHAT IS THE BEST POST EXPOSURE
PROPHYLAXSIS?
331. OLD MAN PRESENTING WITH
1-two drugs (antiretroviral) for 2 wks
CHOLECYSTITIS NO RESPONSE TO 2-three drugs for 2 wks
ANTIBIOTICS 3-two drugs for 4 wks
ERCP with stent 4-three drugs for 4 wks
PTC with STENT
CHOLECYSTOTOMY. IV. ANTIBIOTIC 337. as regarding AIDS WHAT IS
CORRECT
332. Staph epidermis 1-lung is affected in 2/3 of the cases when the
Common in artificial valve syndrome started
HIV. EYE. Infection 2-PCP has an acute onset and rapid course
CMV 3-BAL and biopsy diagnose about 50% of PCP
4-most common mycobacterium is M YEPSI
333. OLD WOMAN UTI CANDIDA what 5-TB started in later stage than PCP
is the best Treatment?
338. post-cholysectectomy antibiotics is
1-cefazoline+amkacin

SABER internal medicine prometric | P a g e 33


2-cefatraxion+erythromycin+meternidazol b) typhoid
3-amoxil+amikacine+meternidazol c) TB
4-cefatoxin+clindamycin
346. A case immunocompromised HIV
339. Chinese farmer with fever and
,what infection most common:
myalgia 4 days and haematuria one day a)pneumocystis carnii
jaundice 2 days WHAT IS DIAGNOSIS b)TB avium
1-dengue fever
2-leptospira 347. A nurse is exposed to a needle brick
3-yellow fever from HIV+ patient who is suspected also
4-malaria
to have cryptoccocal meningitis
340. pt with conjunctival redness inc LFT
Zidovudine, lamivudine for 4 weeks
and RFT and fever WHAT WILL YOU Zidovudine, lamivudine for 4 weeks
DO (brucellosis) Zidovudine, lamivudine and tenofovir for 8 weeks
1-weil fielx test Zidovudine, lamivudine and tenofovir for 8 weeks
2-ELISA
3-paul bannel NB: when i searched on internet i found that NO 4 is
4-haemaglutinin more right

341. vancomycin can used in all except 348. The most common cause of ocular
1-stap aureus manifestations in HIV
2- staph epididmus CMV
3- strept fecalis
4-bactiroid F 349. Patient with dysuria and frequency
5-c difficile
and mild suprapubic tenderness
342. HIV patient with chronic diarrhea urine analysis shows pus cells 30 cell,
and CD4 50 and oocyst seen what to the most common organism is :
give Enterobacter fecalis
Escherichia coli
Nitazoxide (supportive I think)
Metronidazole Borrelia melitinesis

350. farmer had injured with sickle


343. Common in artificial valve
object he had tetanus toxoid 15 years
344. Alkaline urine the cause of infection ago .ttt
is 1-manage the wound and give tt toxicoid
2------------------------------ + tt antiglobulin
proteus

351. Chancroid is caused by:


345. A case of fever with low back pain
a) Chlamydia trichomatis
and sweating and splenomegaly and not b) Haemophillus ducreyi
CMV c) Gardenella vaginalis
a) brucllosis d) Calymatobacterium granulomatis

SABER revision for IM prometric exam| P a g e 34


b- wiles dieaeses
352. All of the following are poor c-……. D-……..
prognostic factors in acute pancreatitis, 358. patient undergo cholycystectomy

except: antibiotic given


a) Age >55years a-cephalospor+cipro+metronidazole
b) Urea > 45 mg\dl b-gentamycin+amikane+cipro
c) Leukocytosis d-cipro+metronidazole
d) Glucose>10mmol\l
e) Serum amylase >1600 IU\L 359. IGRA used for
Mantoux
353. Drug of choice in acute cystitis is: Latent TB
a) Trimethoprim-sulfamethoxazole Active TB
b) Ceftriaxone Monitor RX
c) Ciprofloxacin 500mg/12hourly for 3 days
d) Nitrofurantoin 360. HIV+ Tongue leukoplakia (picture)

354. pat HIV start to take protease CDC 300 class as


A3
inhibitor e AZT e CD4 120 after 4week B2
developed fever e generalized C3
lymphadenopathy what the cause
361. complicated malaria jundice +
a- Side effect from protes inhibitor
b- Immune inflammatory mediated complex splenomegaly+anaemia Rx
c- Acute lymphtic leukaemia Oral Chloroquine
d- Failure of therpy IM Chloroquine
IV artesunate
Fansidar
355. pat HIV e CD4 50 e fever and chest
infection (pneumonia) sputum culture 362. A picture of prodroma of flu like
acide fast bacilia The organism is illness then jaundice , epistaxis and
A-mycobacterium avium haematuria in young Saudi boy
b- mycobacterium bovies Investigation of choice

356. 35y pat e fever ,jaundice,hematuria 363. Complications of varicella except


e picture of peripheral blood film show Pneumonia
intracellular inculuetion (malaria) best Pericarditis
Cerebellar ataxia
maneg Encephalitis
a-oral chloroquine 364. Patient with papulovesicle inguinal
b-iv chloroquine c-………. D-………..
area, tendet LNs, treatment?
Azithromycin
357. pat e fever for one week 2day Ofloxacin
jundies 1day heamaturia D
a-Dengue fever

SABER internal medicine prometric | P a g e 35


365. Diarrhea not treated by: all true
except;
Yersinia entero (-ve) organism
Reactive assymetrical polyarthritis of big joints
Uveitis, mesentric lymphadenopathy
Diagnosed by culture, serology, titre > 1/160*

366. HIV, eye infection is common by:


toxoplasma
EBV
Herpes zoster
CMV*

367. Leishmania symptoms all except:


Epistaxis, spleenomegaly
Leg ulcet
Chest pain
Disfiguring facial ulcer
Cough, dysphagia*

368. Immuno-compromised patient,


vaccine all except:
HBV
Influenza
Polio*
Pneumococci

369. Malaria all true except


Quinine precipitate malaria
Disease manifested if affected RBCs > 51
Hepatosplenomegaly
Rash and lymphadenopathy*

370. Lymphogranuloma venerium all except:


Inguinal LNs and ulceration
Chlamydia trachomatis servor L2
Diagnosed by inclusion bodies in cytoplasm
Chancroid has no role facilitate spread of HIV*

371. Blood film: (malaria) c/o fever,


shivering, ttt?
1 IV artesunate
2 IV chloroquine
3 Oral chloroquine
4

SABER revision for IM prometric exam| P a g e 36


RHEUMATOLOGY
· microscopic polyangiitis, positive in 50-75%
· Churg-Strauss syndrome, positive in 60%
· Wegener's granulomatosis, positive in 25%
Other causes of positive ANCA (usually pANCA)
372. Regarding the ankylosing · inflammatory bowel disease (UC > Crohn's)
spondylitis, all of the following are true · connective tissue disorders: RA, SLE, Sjogren's
· autoimmune hepatitis
,except:
a) Associated with HLA -B27
379. Risk of osteoporosis
b) Exercise increase pain
c) Male more affected than female 1-alcohol
d) Anterior uveitis 2-smoking
3-CST treatment
4-all of above
373. Systemic sclerosis scenario to
5-none of above
diagnose
380. long scenario about a female with
374. osteoarthritis scenario to diagnose repeated fracture, WHAT IS
SIGNIFICANT FOR DIAGNOSIS OF
375. A Q about RA what is true ?
OSTEOMALACIA
Symmetrical
Morning stiffness 1-dec ca,dec po4
2-dec ca,inc po4
3-inc ca ,inc po4
376. OLD LADY HYPERTENSION
4-inc ca ,inc po4
DIABETES THYROIDECTOMY BACK
PAIN 381. Long scenario about female has joint
Osteoporosis pain ,smooth tight skin +ve anti SCL 70
Osteomalacia
has renal impairment cr 170 BP IS

377. A CASE OF WAGNER HOW TO high WHAT IS THE BEST FOR

DIAGNOSE? DECREASE BP IN THIS PATIENT?


LUNG BIOPSY 1-glucocorticoide
2-NSAID
3-ACEI
4- B blocker
378. SCLERODERMA
MORPHOEA presents first xxx
382. Scenario about male patient young
CREST syndrome associated with progressive
course xxx with oral ulcer and arthritis what is
diagnosis:
pANCA present In All except
Autoimmune hepatitis Behçet’s disease
pANCA
· most common target is myeloperoxidase (MPO) 383. A case of Rheumatoid arthritis with
· cannot use level of pANCA to monitor disease swelling behind the Knee joint ,the best
activity
· associated with immune crescentic invest.
glomerulonephritis (positive in c. 80% of patients) a)venogram of the cuff

SABER internal medicine prometric | P a g e 37


b)analysis of the content 391. Antiphospholipid syndrome, all
US
except:
384. A case of arthritis in several joints AD
Livedo reticularis
,what favour it is RA: Recurrent spontaneous abortion*
a) Diffuse similarity Throbocytosis*
b) Taking the big joints
392. A case of fever rash, arthritis,
385. Old male with knee swelling hot and
diagnosis?
tender no history of prolonged fever Rheumatic fever*
,aspiration clear fluid WBC 2000 Rheumatoid arthritis
SLE
culture –ve ? rheumatoid septic OA H.S purpura
gouty

386. Regarding the ankylosing


spondylitis, all of the following are true
,except :
Associated with HLA -B27
Exercise increase pain
Male more affected than female
Anterior uveitis

387. Behçet’s syndrome all except:


Hx of oligoarthritis
Congunctivitis
Mouth ulcers
Genital ulcers
Common in females*

388. RA all true except:


Morning stiffness
Destruction of articular cartilage
HLA DR4
S.C nodules
Equal sex*

389. Renal in HSP


390. Ms all true excepts
Polygenic and gene for tnf Alpha as well as HLA
MRI brain = multiple high signal lesion in paraventral
area
CSF = oligoclonal band
Silent lesion up to 30%, axonal damage*

SABER revision for IM prometric exam| P a g e 38


RENAL
a) Primary hyperparathyroidism
b) Women on longterm analgesics
c) Recurrent urinary tract infections
d) Sickle cell anemia

393. Pre-Renal Failure: 399. Patient with lupus nephritis, which


a. Casts of the statements is true?
b. Urine Osm <400
a) Step 1 and 2 nephritis require no treatment
C. Urine Na <20 mmol/L
b) Stage 3 is treated with aggressive therapy
d. Decreased water excretion
c) Stage 5 has poor prognosis
d) Stage 2 require steroid treatment
394. Polycystic renal disease is:
a. AD 400. Dark urine is present in all, except:
b. AS
a) Gilbert syndrome
C. Sex-linked dominant
b) Porphyria
d. Sex-linked recessive
c) Rifampicin therapy
e. Autosomal dominant combined with...
d) Paroxysmal nocturnal haemoglobinurea

395. Patient with Hx of severe


401. Primary polydipsia findings
hypertension, normal creatinine, 4g Reduced P. osmolality & reduced urine Na
protein 24 hrs. right kidney 16cm &
left kidney 7cm with... suggesting of left 402. in MGD define primary responder
Complete remission after single course steroid
renal artery stenosis. Next investigation: Partial
a. Bilateral ... renal angiography Multiple courses
b. Right percutanious biopsy Spontaneous remission
C. Left precutanious biopsy
d. Right open surgical biopsy 403. Scenario of APKD eFHx of brain
e. Bilateral renal vein determination
hage what is true regarding the
396. Urine analysis will show all except: condition
a) Handling phosphate. RF should be anticipated
b) Specific gravity. MRI screening is not necessary for relative
c) Concentrating capacity. HTN should be Rx e diuretics
d) Protein in urine.
404. causes of papillary necrosis
397. In acute renal failure, all is true NSAID
except: Sickle cell
DM
a) Phosphatemia. Hyperparathyroidism
b) Uremia.
c) Acid phosphate increases.
d) K+ increases. 405. Management of hypercalcemia
Fluid
Calcitonin
398. Papillary necrosis is present in all of
Bisphosphonate
the following conditions ,except: Diuretics

SABER internal medicine prometric | P a g e 39


1.012.[1] Isosthenuria reflects renal tubular
damage/failure of renal medullary function.)
406. Patient on regular Peritoneal
dialysis 200/day , urine output 50/day
, gained weight 413. Treatment of diffuse proliferative
Increase dose of diuretic glomerulonephritis
Add 1.5% dextrose to peritoneal diasylate Cortisone
Prolong time of dwelling? Cortisone with cyclophosphamide
Azathioprin
407. Patient came with renal failure ,
414. WHAT IS THE MOST COMMON
U/S show shrunken kidney:
CAUSE OF DEATH IN ACUTE RENAL
Chronic glomerulonephritis
Polycystic kidney FAILURE
Hyperkalemia
acute pulmonary edema
408. Patient came with renal
impairment , hematuria , U/S show
415. Which of these is characteristic for
normal kidney, what next:
ATN
IV pyelography
1-U/P urea >20:1
CT abdomen
2-U/P creatinin >10:1
MRI abdomen
3-U/P osmolarity >320
DTPA scan
4-urine NA =12 meq
5- SP gravity 1025
409. Renal failure hb=6, normocytic
normochromic, cause of anemia: 416. patient with dialysis C/O symptoms
Decreased erythropoitin of carpal tunnel synd. WHAT IS THE
CAUSE
410. Nehrotic syndrome, what not true 1-deposation of B2 microglobin
Allergy
Gross hematuria 417. hyaline cast with :
Edema Normal finding in urine

411. ESRD pain in left arm upto little


418. iso-osmolar urine in
finger, cause:
Acute tubular necrosis
B2 macroglobulin deposition Renal failure

412. ISO osmolar urine Cause 419. 2-What is the antibiotic


Pre renal Glomerulonephritis contraindicated for UTI in pregnancy
I think DI or ATN Ceftrioxone
(Isosthenuria refers to the excretion of urine Amoxycillin
whose specific gravity (concentration) is neither Nitrofurantoin
greater (more concentrated) nor less (more dilute) ciprofloxacin
than that of protein-free plasma, typically 1.008-

SABER revision for IM prometric exam| P a g e 40


420. A case about APKD 425. all are nephritic s except
Nil disease
RF is anticipated diabetic nephropathy
MPGN
421. Patient with dysuria and frequency
426. Q about a case membranous GN ttt
and mild suprapubic tenderness
………………………… Corticosteroids: prevention of
urine analysis shows yeast, treatment progression to renal failure without improvement
Ketoconazole in proteinuria.
Fluconazole Chlorambucil
…..conazole Cyclosporine
Amphotercin B Tacrolimus
NB: I think it is the right Cyclophosphamide
Mycophenolate mofetil
422. A child is diagnosed with nephrotic
syndrome, what is the least 427. 12 y patient e gaine

complication to occur with this wight HTN oliguria elevated renal


function renal biopsy diffuse
condition
proliferative GN ttt
ARF …………………………………………supportive resolve after 2-
Hypertension 4 weeks…….
Allergic manifestations 428. 18y patient with heavy proteinuria,
Gross heamaturia
hyperlipidemia renal biopsy minimal
N.B: I answered it as Allergic manifestations because I changes GN what true
do not know a case of nephrotic syndrome with a- ARF most likely occur in acute phase
allergy. beside, NS occure with gross heamaturia in a b- ttt by steroid and cyclophosphamide
minority of IgAN but may answer was considerd c- Most of cases show spontaneous remission
wrong d-………..

423. Diffuse proliferative lupus nephritis


429. A case e , elevated uric
is treated by:
acid mechanism of renal affection
Prednisolone only
Steroids and cyclosporine a-urate nephropthy
MMF b-…c-………d-………
Azathioprine
430. ttt of lupus nephritis e heavy
424. Patient with NH lymphoma after proteinuria
chemotharpy >>> uric acid 9 creat high ……………….MMF is safer than
urea high ? cyclophosphamide with corticosteroids.
1-chemtherapy SE to kidney
2-tumour lysis syndrome 431. D ,HTN e proteinuria maneg
3-lymphoma affect kidney a-lasertan
b-hydralzin

SABER internal medicine prometric | P a g e 41


c…………… 438. Diarrhea followed by renal failure
Hemolytic uremic syndrome
432. Q aboute causes of papillary
necroses 439. ttt of diabetic nephropathy
Causes: SODAT Losartan*
· chronic analgesia use Lasix
· sickle cell disease Prazocin
· TB Propranolol
· acute pyelonephritis
· diabetes mellitus 440. US showing bilateral small kidneys?
Features
AGN
· fever, loin pain, haematuria
ATN
· IVU - papillary necrosis with renal scarring - 'cup &
Chronic GN *
spill'
Pre-renal failure
441. Isosmolar urine
433. case of lupuse nephrites specific for
ATN
diagnosis GN
a- ANA RF*
b- AntiDs DNA Hepatorenal syndrome
c- RNP 442. Uremic symptoms all except:
d- …..(I thinl anti smith then ana) High K
Hemorrhagic pericarditis
434. Typical scenario of post strep. u/s Decreased LFTs*
Oliguria
done normal kidneys GN what to do
LL edema
next? Persistant extracellular fluid expansion during
Abd. X-ray diuretics treatment
CT 443. Renal transplantation
IVP
DTPA
444. Renal biopsy contraindication in:
Renal artery stenosis
435. Management of hypercalcaemia
DBP > 120*
except: GN
Fluid ARF
Calcitonin
Bisphosphonate 445. GN one is true
Diuretics Sonar differentiate between acute and chronic
Biopsy is important for diagnosis*
436. Minimal change GN
Podocytes 446. Microalbuminurea except?
Morning midstream*
437. Diffuse proliferative GN Early sign of diabetic nephropathy
Steroids
Mycophenolate 447. APKD
Steroids + cyclophosphamide Renal failure is anticipated*

SABER revision for IM prometric exam| P a g e 42


Renal biopsy for diagnosis
Carotid angiography

448. Foamy urine

449. Polycystic kidney


1 gross hematuria
2 most patients are hypertensive
3 Berry’s aneurysm almost present in all patients
Transplantation is CI

450. A picture of prodroma of flu like


illness then jaundice, epistaxis and
hematuria in young Saudi boy
Investigation of choice?

SABER internal medicine prometric | P a g e 43


ENDOCRINE
1. Intravenous fluid hydration and electrolyte
homeostasis

o Phosphate, magnesium, and calcium are not


451. a patient (known case of DM) replaced routinely, but a patient symptomatic
with tetany requires replacement therapy.
presented to you with diabetic foot
 Correction of hyperglycemia
(infection) the antibiotic o All patients with HNS require treatment with
Antibiotics intravenous insulin; however, immediate
Appropriate monotherapy for cellulitis includes treatment with insulin is contraindicated in the
cefazolin or clindamycin. Although gram-negative initial management of patients with HNS. The
organisms are the unusual causes of cellulitis, even in osmotic pressure that glucose exerts within the
diabetes, if they are suspected, a fluoroquinolone vascular space contributes to the maintenance of
(eg, levofloxacin) may be used in conjunction with circulating volume in these severely dehydrated
clindamycin. patients. Institution of insulin therapy drives
glucose, potassium, and water into cells. This
452. In DKA, all the following are true results in circulatory collapse if fluid has not been
replaced first.
except:  Begin a continuous insulin infusion of 0.1
a. Increase of corticosteroid, glucagon and GH. U/kg/h.
b. Body water deficit is 4 to 6 L
For blood glucose concentrations of less than 250
453. A 14 years old boy with type 1 mg/dL, decrease the insulin infusion rate by 0.5 U/h.

D.M. presented in coma. His blood


454. A 45 year old presented with
glucose level is 33 mmol/l. Na is 142
polyurea, urine analysis showed
mmol/l, K is 5.5 mmol/l, bicarb is 10
glucosurea & -ve ketone FBS l4mmoI.
mmol/l. the following are true except:
What is the best management of this
a. The initial Rx. Should be l.V. normal saline 3 L/hour
for 1-2 hours patient?
b. IV. insulin loading dose 1 u/kg is necessary. (0.1 a. Intermediate IM insulin till stable
U/kg/h is the true) b. NPH or Lent insulin 30mg then diet
c. IV. Na bicarbonate could be given if pH is 7 or less. C. Sulphonylurea
d. Hyprephosphatemia can occur during treament. d. Diabetic diet only
e. Hyperchloremia can occur during treatment e. Metformin

Hyperosmolar Coma: Treatment & Medication 455. A 36 years old female with FBS =
All patients with HNS require hospitalization, and
most should be admitted directly to the intensive 14 mmol & glucosuria, without ketones
care unit (ICU). When available, an endocrinologist in urine, the treatment is:
should direct the care of these patients. The main a) Intermittent I.M. insulin NPH.
goals of treatment are to b) Salphonylurea + diabetic diet.
(1) vigorously rehydrate the patient while c) Diabetic diet only.
maintaining electrolyte homeostasis; d) Metformin.
(2) correct hyperglycemia;
(3) treat underlying diseases; and
(4) Monitor and assist cardiovascular, pulmonary,
renal, and CNS function.
SABER revision for IM prometric exam| P a g e 44
456. A 30 years old teacher complaining 2) A random plasma glucose concentration ≥ 11.1
mmol/L (200 mg/dL) accompanied by classic
of excessive water drinking and symptoms of DM (polyuria, polydipsia, weight
frequency of urination, 0/B Normal. loss) is sufficient for the diagnosis of DM.
You suspect DM and request FBS = 6.8 Oral glucose tolerance testing, although still a valid
.the Dx is: means for diagnosing DM, is not recommended as
a. DM part of routine care.
b. DI
c. Impaired fasting glucose 457. 60 years old male complaining of
d. NLbloodsugar
decreased libido , decreased ejaculation,
e. Impaired glucose tolerance
Glucose tolerance is classified into three categories FBS = 6.5 mmol, increased prolactin,
based on the FPG : Normal FSH and LH, your opinion is
(1) FPG ≤ 5.5 mmol/L (100 mg/dL) is considered
a. Measure Testosterone level
normal;
b. He has DM
(2) FPG = 5.6–6.9 mmol/L (100–125 mg/dL) is defined
c. Do CT of head
as IFG; and
d. He has Normal Fasting Blood sugar
(3) FPG ≥ 7.0 mmol/L (126 mg/dL) warrants the

458. A 46-year-old man, a known


case of diabetes for the last 5
months. He is maintained on
Metformin 850 mg Po TID, diet
control and used to walk daily for
30 minutes. On examination :
unremarkable. Some investigations
diagnosis of DM.
show the following: FBS 7.4 mmol/L ,2
Based on the oral glucose tolerance test (OGTT),
hr PP 8.6 mmol/L ,HbA1c 6.6% ,Total
Impaired glucose tolerance (IGT) is defined as Cholesterol 5.98 mmol/L ,HDLC 0.92
plasma glucose levels between 7.8 and 11.1 mmol/L mmol/L ,LDLC 3.88 mmolIL ,
(140 and 199 mg/dL) and
diabetes is defined as a glucose > 11.1 mmol/L (200 Triglycerides 2.84 mmolIL (0.34-2.27)
mg/dL) 2 h after a 75-g oral glucose load. ,Based on evidence, the following

Some individuals have both IFG and IGT. Individuals concerning his management is true:
with IFG and/or IGT, recently designated pre-diabetes 1. The goal of management is to lower the
by the American Diabetes Association (ADA), are at triglycerides first.
substantial risk for developing type 2 DM (25–40% 2. The goal of management is to reduce the HbA1c.
risk over the next 5 years) and have an increased risk 3. The drug of choice to reach the goal is Fibrates.
of cardiovascular disease. 4. The goal of management is LDLC ≤ 2.6 mmol/L.
5. The goal of management is total cholesterol ≤ 5.2
The current criteria for the diagnosis of DM mmol/L.
emphasize that
1) the FPG is the most reliable and convenient test
for identifying DM in asymptomatic individuals.

SABER internal medicine prometric | P a g e 45


According to guidelines of the ADA and the 2. FPG ≥ 7.0 mmol/L plus 2 h-post 75 gm glucose ≥
American Heart Association, the target lipid values 11.1 mmol/L is diagnostic of diabetes mellitus.
in diabetic individuals (age >40 years) without 3. FPG ≤ 5.5 mmol/L = normal fasting glucose.
cardiovascular disease should be: 4. FPG ≥ 7.0 mmol/L = provisional diagnosis of
1) LDL < 2.6 mmol/L (100 mg/dL); diabetes mellitus and must be confirmed in
2) HDL > 1.1 mmol/L (40 mg/dL) in men and >1.38 another setting in asymptomatic patient.
mmol/L (50 mg/dL) in women; and 5. 2-h post 75 gm glucose ≥ 7.6 mmol/L and < 11.1
3) triglycerides < 1.7 mmol/L (150 mg/dL). mmol/L = impaired glucose tolerance.
Impaired glucose tolerance (IGT) is defined as
Fibrates have some efficacy and should be considered plasma glucose levels between 7.8 and 11.1 mmol/L
when the HDL is low in the setting of a mild elevation (140 and 199 mg/dL)
of the LDL.
460. In DKA, use: both wrong
459. Regarding the criteria of the a) Short and intermediate acting insulin.
diagnosis of diabetes mellitus, the b) Long acting insulin.

following are true except:


461. a 70 yr old male, suddenly felt
1. Symptomatic patient plus casual plasma glucose
≥7.6 mmol/L is diagnostic of diabetes mellitus. down & he is diabetic, it could be:

SABER revision for IM prometric exam| P a g e 46


a) May be the patient is hypertensive and he
developed sudden rising BP 466. 25 years old female underwent
b) He might forgot his oral hypoglycemic agent dose
c) Sudden ICH which rise his ICP surgery for thyroid, in the recovery
room she developed fever 102F , pulse
462. 42 y/o female presented with 5 120\min, B.P 160\100 , her
months Hx of N, V & malaise. Na is low, management will include all of the
Cr is high, Glucose is 2.7 mmol/L & following drugs, except:
HCo3 is 10. a) Propranolol
a- hypothyroidism. b) Oral or IV iodine
b- Addison’s disease. c) High doses of aspirin
c- hypervolemia because of the vomiting. d) I.V steroids
d- pheochromocytoma.
e- SIADH. 467. 30 years old man present with loss
of consciousness in ER, his blood glucose
463. In a patient with elevated serum
was 102mmol/L, C-peptide level is
level of calcium without hypocalciuria,
absent, insulin concentration 400 IU
which of the following tests is almost
(normal range :20-200). What is the
always diagnostic of primary
diagnosis?
hyperparathyroidism:
a) Islet cell hyperplasia
a. Elevated serum level of ionized calcium. b) Exogenous insulin
b. Elevated serum level of chloride and decreased c) Oral hypoglycemic agent
serum phosphorus. d) Insulinoma
c. Elevated serum level of intact parathyroid
hormone (PTH).
d. Elevated 24-hour urine calcium clearance. 468. Regarding type 2 DM , which of the
e. Elevated urinary level of cyclic AMP. following statement is true?
a) All type 2 diabetic patients have diabetic
464. Primary hyperparathyroidism is nephropathy at the time of diagnosis
b) Type 2 diabetic retinopathy definitely have
confirmed by demonstration of an
nephropathy at the same time
inappropriately high PTH level for the c) There is strong co-relation between kimmelsteil
degree of hypercalcemia. Wilson and retinopathy
d) Urine dipstick can detect albuminuria >100 mg/24
hr
465. Some patients develop
hypoparathyroidism after thyroid or 469. pruritus
parathyroid operations. What is the Occurs in both hypo &hyperthyroidism
Common symptoms of DM
treatment for hypoparathyroidism:
a. Oral 1,25-vitamin D and calcium.
b. Transplantation of fetal parathyroid tissue. 470. DKA: PH 7.09 first to correct
c. Intramuscular PTH injection.
Acidosis with HCO3
d. Reoperation to remove the thymus.
Dehydration with NS
e. Oral phosphate binders.
Hyperglycemia with INSULIN

SABER internal medicine prometric | P a g e 47


Electrolytes
1-DM type1
471. Rx of hyperthyroidism in pregnancy 2-type II DM
3- repeat the test
Antithyroid drugs
4- Not diabetic
Surgery
Radioiodine
Propranolol 479. patient newly discovered DM ,he is
obese,RBS 240mg/dl which drug should
472. Myoxdema coma Rx RR 6
be taken
Ivthyroxine +iv dextrose+ iv steroid+mechanical
1- glibencamide
ventilation
2- sitaglibitin
3- metformin
473. Addison test to confirm 4- insulin
Cortisol 5- exantide
ACTH
Cortisol response to costyrpin 480. Patient with hypertension K=2.1 :
Na 128 is consistent with 1ry hyperaldosteronism
474. patient with feature of cushing Rennin activity increase in hyperaldosteronism
syndrome given 1 mg dexamethasone High salt will worsen hypokalemia

overnight cortisol measured 8 a.m


481. An old male routine checkup
398 what to do next
presenting with very high alkaline
Low dose 0.4 mg dexa 4 time 6 hourly
24 hour urine free cortisol phosphatase, normal Ca & ph

475. Cons syndrome common cause Paget disease


Osteomalacia
Adrenal adenoma
Bilateral adrenal hyperplasia
482. YOUNG BOY WITH polydipsia AND
476. DM with gastroparesisginven POLYURIA CT SHOWED
metoclopramide and erythromycin , no CALCIFICATION of pituitary
response what to give(not fully HISIOCYTOSIS X
CRANIOPHARYNGEOMA
memorized) TB PITUITARY
Omeprazole SYPHILIS
Metronidazole
Gastrostomy feeding 483. Psychogenic polydipsia
Low Na serum & urine
477. pt for routine check up discovered High Na serum & urine
he has RBS is high , OGT done he is Low Na serum high urine
High Na serum low urine
478. obese and 45 ys old, This gentleman
484. Diagnosis of Acromegaly
has
Growth hormine
Time 0 1hr 2hr leve of IGF
130 180 250
SABER revision for IM prometric exam| P a g e 48
GLOMERULOSCLEROSIS ARE
*Growth hormone (GH) levels vary during the day
and are therefore not diagnostic. The ASOCITED
definitive test is the oral glucose tolerance (OGTT)
with serial GH measurements.
*Serum IGF-1 may also be used as a screening test 490. MACROPROLACTINOMA WITH NO
and is sometimes used to monitor disease PRESSURE SYMPTOMS
*Oral glucose tolerance test
· in normal patients GH is suppressed to < 2 mu/L Bromocreptine and monitor clinically and laboratory
with hyperglycaemia
· in acromegaly there is no suppression of GH
491. female pt has infertility
· may also demonstrate impaired glucose tolerance
which is associated with acromegaly ,galactorrehea ,MRI reveal adenoma
*A pituitary MRI may demonstrate a pituitary tumour 6mm prolactin is 350 WHAT IS YOUR
TREATMENT?
485. HYPOTHYROIDISM ALL EXCEPT 1-bromocreptine
2-surgical
3-somatostatine
Diarrhea or clubbing what ever
4-observation

1.DIABETC PATIENT WHAT IS THE BEST


492. Regarding Amiodarone induced
ANTIHYPERTENSIVE?
hyperthyroidism WHAT IS WRONG?
ACE inhibitor
1-the cause of the symptoms is iodine content of the
drug
486. DIABETC PATIENT with 2-surgical is an option of treatment
proteinuria(no ACE inhibitor) 3-radio iodine is an option of treatment
4-treatment with anti-thyroid
ARBS (losartan)

493. diabetic pt his albumin in urine 600


487. LIDDLE SYNDROME
Hypokalemia mg WHAT IS THE BEST FOR HIM
Metabolic Alkalosis 1-ACEi
2-albumin
488. A MIDDLE AGED MAN WITH 3-B blockers
4- Add anti diabetic drug to his medications
HYPERTENSION AND MUSCLE
WEAKNESS,HYPERNATREMUIA,METAB 494. A patient takes haloperidol is
OLIC ALKALOSIS presented with leathergy and coma and
Cushing Temp is 40 and CK is 1000
Adenoma if he said Conns
Neuroleptic malignant syndrome
Bilateral adrenal hyperplasia
Malignant hyperthermia
Best control for IDDM
489. DIABETC RETINOPATHY Basal and bolus insulin
AND KIMMELSTIEL–WILSON
SYNDROME, OR NODULAR DIABETIC

SABER internal medicine prometric | P a g e 49


495. patient presented with
NB: you notice the strange choice number 4 ?
hypoglyceamia after oral How can I change and increase the dose of ateno in
hypoglyceamics the same time?
Sulphonylurea
500. A patient recently underwent

496. The initial treatment of DM subtotal thyroidectomy presented with


agitation, fever, dyspnea and
tachycardia
Metformin
Insulin hydrocortisone,propranolol, propylthiouracil and
iodide
497. What is the treatment of addisonian NB: carbimazole is more preferred than
disease propylthiouracil but this is the most suitable answer
IVI of normal saline and D5%
Hydrocortisone now and every 8 hours
Optain sample for testing of serum cortisone and 501. Female pt on oral medication of
ACTH
hyperthyroidism she had fever
All of the above
tachycardia drowsiness , she had gall
498. A case of lethargy, impotence and bladder surgery last week , what is the
loss of lipido with bitemporal headache treatment now:
Cushing syndrome 1-carbimazole
pituitary tumor 2-propylthiouracil
3-radioactive iodine
499. A case about a patient that has DM 4-surgery
on oral treatment of metformin 500
502. Cause of death in
mg twice and atenolol 50 mg four
Pheochromocytoma
times daily and frusemid 20 mg and arrhythmia
ator 10 mg daily has BP of 130/90 MI
CVA
and fasting blood glucose level of 140
All
and Chlesterol 230 (normal less than
200) and high LDL and low HDL and 503. Regarding type 2 DM , which of the
high triglycerides (only slight elevation following statement is true ?
in all but I do not exactly remember a) All type 2 diabetic patients have diabetic
nephropathy at the time of diagnosis
their values) and BMI is 30
b) b) Type 2 diabetic retinopathy definitely have
What to do for this patient? nephropathy at the same time
Increase the dose of ator c) c) There is strong co-relation between
Increase the dose of ator and metformine kimmelsteil Wilson and retinopathy
change ateno with another agent
d) d) Urine dipstick can detect albuminuria >100
change ateno and frusemide and increase the dose of
mg/24 hr
ateno and metformine

SABER revision for IM prometric exam| P a g e 50


504. patient with feature of cushing Klinfelter syndrome
Asharman syndrome
syndrome given 1 mg dexamethasone
overnight cortisol measured 8 a.m 508. A lady with amenorrhea, 3 months
398 what to do next progesterone treatment for 5 days = no
Low dose 0.4 mg dexa 4 time 6 hourly withdrawal bleed, increased LH and
24 hour urine free cortisol
FSH, normal prolactin, Diagnosis?
Tests to confirm Cushing's syndrome POS
The two most commonly used tests are: Primary ovarian failure*
· overnight dexamethasone suppression test (most Pregnancy anovulation
sensitive) Pituitary disease
· 24 hr urinary free cortisol
509. POS except:
Localisation tests
Minor increase of androgen
The first-line localisation is 9am and midnight plasma
Mild increase of prolactin, high ldh
ACTH (and cortisol) levels. If ACTH is
Obese with second amenorrhea
suppressed then a non-ACTH dependent cause is
High FSH, LH
likely such as an adrenal adenoma
Non of above
High-dose dexamethasone suppression test
· if pituitary source then cortisol suppressed
· if ectopic/adrenal then no change in cortisol 510. Antithyroid drugs all except
CRH stimulation Antithyroid give immediate effect
· if pituitary source then cortisol rises Propylthiouracil is not CI in pregnancy
· if ectopic/adrenal then no change in cortisol Antithyroid should start with large dose
Petrosal sinus sampling of ACTH may be needed to Eye symptoms do not improved with treatment
differentiate between pituitary and ectopic 511. Barters syndrome all except:
ACTH secretion Normotensive
Hypokalemia
505. Conn’s syndrome common cause Metabolic alkalosis
Adrenal adenoma Metabolic acidosis*
Bilateral adrenal hyperplasia

506. female e glactorrhea, amenorrhea, 512. Best benefit in altetnating


increasesd prolactin MRI pituitary corticostetoids to avoid:
adenoma no pressure symptoms what Growth retardation
Osteoprosis
next
513. Patient with SLE, the best
Surgery
Octreotide sunscreen?
Cabergoline Monobenz
TSH 514. Tiranium oxide
515. Causes of hypercalcemia all except
507. A case of hypo GRH Associated with
Familial hypercalcuria
anosmia Cushing dse*
Kallman syndrome? Sarcoidosis
Turner syndrome Primary hypoparathyroidism

SABER internal medicine prometric | P a g e 51


516. Aldosteronism occurs in all except
Hemorrhage
Hyperkalemia
Carbonic anhydrase inhibitor administration*
ICU
517. Difference between primary and
522. Regarding to the shock
secondar6 hyperaldosteronism? 1-in septic shock CO increased
2-in cardiogenic shock PCWP increased
3-in hypovolemic shock PH >7.3
518. Hyperlipidemia complicate for all
except: 523. Patient diagnosed as dengue fever
Hypothyroidism
BP 90/60 WBC 3000 HB 8 g/dl
Nephrotic syndrome
Estrogen treatment PLATLET 80.000 WHAT IS YOR
HTN* TREATMENT
1-1L crystalloid
519. Hypothyroid patient, what is not 2-Granulocyte transfusion
suggested 3-inotropes
Diarrhea 4-platelets
Menstrual irregularity
Bradycardia 524. To confirm it is septic shock
4 1-inc CO and dec PVR
2-S.lactate >16
520. Treatment of hyperchylomicronemia? 3-PH <7.1
Gemfibrozil, fat free diet* 4-PCWP>12
Fibrates
Statins 525. 82 y/o female presented to ER in
confusion with hypotension. BP was
521. Pheochromocytoma which patient
70/20, P=l6O/min, rectal T= 37.7°C.
need follow up
The most likely of the following would
+ve FHx of Pheochromocytoma
Patient with HTN and hyperglycemia suggest sepsis as a cause of hypotension
3 is:
All of the above
a- Low systemic vascular resistance & high cardiac
output.
b- High systemic vascular resistance & low cardiac
output.
c- Pulmonary capillary wedge pressure less than 26.
d-PH is less than 7.2.
e- Serum lactate dehydrogenase more than 22.

SABER revision for IM prometric exam| P a g e 52


DERMATOLOGY
b) gas gangrene
c)…………..

532. All cause non palpable purpura


526. Which of the following sign is the
except
feature of both erythema multiform
IE
minor and erythema multiform major HSP
(Steven Johnson syndrome) ? Fever
a) Target or iris type lesion
b) Mucus membrane involvement 533. In urticarial vasculitis appear in
c) Conjunctiva involvement Within 24 hrs
d) Involvement of esophagus and palate 48-72 hrs
3-6 months
527. Pyoderma gangrenosum occurs in > 6 months
MM
AML 534. Patient c/o urticaria, used
IBD corticosteroids, no response, what to
ALL
give?

528. all of this condition may have


535. Acne due to steroid therapy, best
subcutaneous nodule EXCEPT
site?
1-sarcodosis
2- R.A Chest
3- R.FEVER Back
4-SLE Face
5- Retier`s dis
536. Function of sebum except:
529. which of these conditions not a Source of vit. E
cause of diffused hyperpigmentation
537. Patient on phynetoin developed
1-pemphegus
2-acanthosis nigrans target lesion, blisters, erosion 50% body
3- underlyning malignancy TEN
SJS
530. vancomycin can used in all except SSSS
1-stap auries EM
2- staph epididmus
3- strept Fecalis 538. Treatment of psoriasis in
4-bactiroid F
pregnancy?
5-C. difficile
Tar
CST
531. A case of skin infection complicated PUVA
by desquamation and marked Isoretinoid
hypotension :
a) staphylococcal toxic shock syndrome

SABER internal medicine prometric | P a g e 53


539. Old woman suffering from painful
gradual plaques increasing in size, h/p
atypical spindle cell, stormiform CD 32
+ve. What is true?
Simple excision
Regional LN metastasis
CD 44 confirm Dx

540. About atopy associated withlens


opacity
5-10%
10-20%
10-15%

541. Acanthosis negricans is associated


with?

542. Treatment of leprosy?

543. Rosacea what is not true?


Increased sebum

544. Side effects of PUVA?

545. Most common site of vitiligo?


Acrofacial

546. Folliculitis decalvans treatment?


Rifambicin

547. Auspitz sign?

SABER revision for IM prometric exam| P a g e 54


METABOLIC
• Cancer
• Nephritic syndrome
• Kwashiorkor

548. When lactic acid accumulates, body 554. In a patient with weight loss, all can
will respond by: be a cause except:
a) Decrease production of bicarbonate a) Thyrotoxicosis.
b) Excrete CO2 from the lungs b) Nephrotic syndrome.
c) Excrete Chloride from the kidneys c) TB.
d) Metabolize lactic acid in the liver d) AS.

549. What is the initial management of 555. The most common cause of
acute hypercalcemia? hypercalcemia in a hospitalized patient
a) Correction of exter-cellular fluid (by adequate
is:
rehydration)
a. Dietary, such as milk-alkali syndrome.
b. Drug related, such as the use of thiazide diuretics.
550. The first step in the management of
c. Granulomatous disease.
acute hypercalcemia should be: d. Cancer.
A. Correction of deficit of extracellular fluid volume. e. Dehydration
B. Hemodialysis.
C. Administration of furosemide. 556. Hyperkalemia is characterized by all
D. Administration of mithramycin.
of the following except:
E. Parathyroidectomy.
a) Nausea and vomiting.
b) Peaked T-waves.
551. All of the following signs or
c) Widened QRS complex.
symptoms are characteristics of an d) Positive Chvostek sign.
extracellular fluid volume deficit except: e) Cardiac arrest in diastole.

means dehydration
557. Normal daily caloric intake is:
A. Dry, sticky oral mucous membranes.
a) 0.3 kcal/kg
B. Decreased body temperature.
b) 1.3kcal/kg
C. Decreased skin turgor.
c) 2.Okcal/kg
D. Apathy.
d) 3.Skcal/kg
E. Tachycardia.
e) 35kcallkg

552. Blood pH
558. Hypokalemia occurs with all except:
a) high after diarrhea
a) Metabolic alkalosis
b) low after vomiting
b) Acute tubular acidosis
c) more in Rt atrium than Lt atrium
c) Chronic diarrhea
d) lower in Rt atrium than Lt ventricle
d) Hyperaldosteronism
e) lower in renal vein than renal artery
e) Furosemide

553. All cause recent loss of weight,


559. An old male routine checkup
except:
presenting with very high alkaline
• AIDS
phosphatase (nothing else)

SABER internal medicine prometric | P a g e 55


567. patient e hyperglycemia ABG
Paget disease acidosis e hyperkalemia D
a- DKA
560. A case in which Fasting morning
phosphate is low 568. Causes of metabolic acidosis e
normal anion gap
1 -Hyperparathyroidism
2 -Congenital rickets · Gastrointestinal bicarbonate loss: diarrhoea,
3- Aluminum ureterosigmoidostomy, fistula
· Renal tubular acidosis
· Drugs: e.g. acetazolamide
561. A case of HYPERKALEMIA
· Ammonium chloride injection
Treatment · Addison's disease)

All except sodium bicarbonate

562. Ph = 7.50 pco2 = 8 po2


= 94, It is a case of :
Pure respiratory alkalosis
Respiratory alkalosis with metabolic acidosis
Respiratory alkalosis , metabolic acidosis and
metabolic alkalosis

563. A case of hypokalemia and HTN


with proximal weakness:
a) Conn’s syndrome
b) Cushing

564. A case of hypercalcemia,:


a) multiple myeloma
b) renal failure
c) breast cancer

565. all causing hypercalcemia except 2ry


hyperpara

566. patient e ascites LL oedema


,jundice e history of 3motion of loose
stoole cbc PLT 70 Na 127 K3.1
cause of disteruped conscious level
a- history of 3motion of loose stool
b- hypokalemia
c- hyponatrimia
d- acidosis

SABER revision for IM prometric exam| P a g e 56


DRUGS
a) Na+
b) K+
c) phosph.
d) none of the above
569. The mechanism of action of ASA:
• Inhibition of the platelet cyclo-oxygenase. 576. All of the following are side effects
• decrease the lipids.
of furasomide except:
a. Hyperkalemia
570. Anticoagulant effect of heparin
b. Hypoglycemia
based on: C. Bronchospasm
A. Alteration of thrombin levels d. Haemolytic anemia
B. Potentiation of antithrombin III e. Pre-renal azotemia
C. Activation of plasmin into plasminogen
D. Inactivation of ionized calcium 577. Digoxin toxicity:
E. Reduction of available factor VII a) Tinnitus
b) Pleural effusion
571. Heparinization includes all except: c) Nausea
-increase PT d) All of the above
-increase PTT e) None of the above
-Anti inflammatory.
- open potential collateral vessels 578. Which of the following is not
associated with phenytoin toxicity?
572. Which drug can be given to G6PD
patient? - Hirsutism
a- ASA - Osteomalacia
b- Sulphonamide - Ataxia
c- Nitrofurantoin - Osteoprosis
d- Chloroquine
579. All of them are renal complications
573. One of the following combination of
of NSAIDs except:
drugs should be avoided: a. Acute renal failure
a) cephaloridine and paracetamol b. Tubular acidosis
b) penicillin and probenecid C. Interstitial nephritis
c) digoxin and levodopa d. Upper GI bleeding
d) sulphamethoxazole and trimethoprim
e) Tetracycline and aluminium hydroxide 580. NSAIDs cause all except:
a. Acute Renal failure
574. All of the following drugs advised to b. Tubular necrosis
be given to elderly pt, except: c. Hypokalemia
d. Interstitial nephritis
a) cimitidine
b) thyroxin
c) digoxin 581. Which one of these drugs is
d) chloropropamide administered orally:
a) Amikacin
575. Furosemide increase excretion of: b) Neomycin

SABER internal medicine prometric | P a g e 57


c) Gentamycin 586. Patient on chemotherapy presented
d) Streptomycin
e) Tobramycin. with fever, all should be done, except:
a) Blood culture
582. Chronic use of steroids will give: b) urine culture
c) aspirin is effective
a) Osteomalacia.
d) broad spectrum antibiotics
b) Myopathies of pelvic girdle
c) Increased risk of breast Ca
d) Hypoglycemia 587. Nitroglycerine cause all of the
following, except:
583. All of the following cause gastric a) increase coronary blood flow.
irritation, except: b) Methemoglobinemia.
c) Venous pooling of blood.
a) Erythromycin.
d) Efficient for 5 mm if taken sublingual.
b) NSAIDS.
e) Lowers arterial blood pressure
c) Sucralfate Anti-ulcer agent, gastric mucosa
protectant.
d) Diclofenac 588. Allopurinol, one is true:
e) Penicillins b) Effective in acute attack of gout.
c) decreases the chance of uric acid stone formation
584. All of the following are anti- in kidneys
d) Salicylates antagonize its action.
arrhythmic drugs, except:
a) Xylocaine 589. Oral anticoagulants:
b) Digoxin
a) can be given to pregnants during 1st trimester
c) Quinidine
b) can be reversed within 6 hours
d) Amiodarone
c) are enhanced by barbiturates
e) Procainamide
d) can not cross blood brain barrier
Group IA : Quinidine sulfate PO ,Quinidine gluconate e) none of the above
PO ,Procainamide IV PO , Disopyramide PO
Group IB : Lidocaine IV , Mexiletine PO 590. Which drug does not cross the
Group IC : Flecainide PO , Propafenone PO
Group II : Metroprolol IV , Esmolol IV placenta:
Group III : Amiodarone PO IV , Ibutilide IV , Dofetilide a) Heparin
PO , Sotalol PO b) Aspirin
Group IV : Verapamil IV PO , Diltiazem IV c) Warfarin
Other : Digoxin IV, PO , Adenosine IV d) Tetracycline
e) Diazepam
585. All can be used for the treatment of
591. all the following are side effect of
acute gout except:
a) Allopurinol thiazide diuretics except:
b) Penicillamine a-has diabetogenic effect
c) Gold salt in severe inflammation b-cause hypocalcemia
d) Paracetamol c-cause hypomagnesimia
e) Indomethacin d-flat curve response
e-cause hypokalemia

SABER revision for IM prometric exam| P a g e 58


592. Regarding β2 blocker all are true Hyperreflexia

except:
600. TCA TOXICITY
a-morning dose effective more than evening dose.
leads to death if exceeds 250 mg
b-long term maintance therapy should be avoided
Needs hemocharcoal
exert most of their effect within first 6 hours
593. All true about cephalosporin use,
except: 601. WHICH DRUG CAUSES COUGH
- The most common side-effect is allergy Lisinopril
- There is a skin test for cephalosporin sensitivity Theophylline
Attempts to develop a skin test for allergy to Diltiazem
cephalosporins have been unsuccessful
602. CURVE FOR DRUG NARROW
594. Which of the following combination
THERAPEUTIC RANGE
is safe: Theophylline
a-alcohol and metronidazole
b-digoxin and amiodarone 603. Absorption of oral drugs
c-warafrin and propanolol
First pass metabolism in liver before circulation
d-furosemide and gentamycin
Enteric coated less absorbed If diarrhea
Migraine decrease absorption
595. Patient brought to the ER with
tricyclic overdose, all of the following 604. ABG showed severe metabolic
statements are false, except: acidosis, what causes:
a) Dose >250mg is dangerous and fatal Hyperventilation
b) Best treatment will be charcoal hyperperfusion in Methanol toxicity
comatose patient
c) Maximum signs and symptoms appear in 6 hours 605. Polymyalgia rheumatica
d) Tricyclic overdose is rare in Europe
Pain in proximal muscles with stiffness
Elevation of cpk mb
596. Drug induced lupus Mild elevation of ESR
Occurs less in slow acetylators
Occurs more in slow acetylators 606. Rheumatoid case what ttt:
Not reversible on stopping the offending drug
methotrexate

597. Define first pass metabolism


607. Rheumatic fever:
598. ethanol intoxication will not cause Erythema multiform ---MARGINATUM
Blurring vision
Huper-reflexia 608. AMIKACIN, TOBRAMYCIN
Hypoglycemia Vestibular toxicity
cochlear toxicity
599. Alcohol toxicity presentation renal toxicity

includes all except:


609. PEG IFN CONTRAINDICATIONS
Hypoglycemia
Hypothermia

SABER internal medicine prometric | P a g e 59


lactation Hypothyroidism
unstable angina Tremors
Ataxia
610. What is true about bioavailability of DI (nephrogenic)

drugs?
619. Concerning CO poisoning WHAT IS
WRONG?
611. Maximum signs and symptoms ocurr
a. Half-life of symptoms from 4 to 6 h
in 6 hours b. Carboxy HB is diagnostic
It is rare in Europe c. If the level >30 seizure and coma
Charcoal used in comatose patient d. hyperbaric O2 is treatment of choice

612. gingival hyperplasia present in


Cyclosporine
Phenytoin
AML
ALL OF ABOVE

613. Name of drug nesritide


Vasoconstrictor
Drug for brain …………..
None of the above

614. Hypothermia
Most of the symptoms and signs of toxcicty occure
within the first 6 hours

615. The mechanism of action of ADH is


increased permeability of the distal tubule to the
water

616. Complication of radiotherapy all


except:
1 retinal detachment
2 diarrhe
3 primary lymphoma of brain

617. Dark urine is present in all, except:


a) Gilbert syndrome
b) Porphyria
c) Rifampicin therapy
d) Paroxysmal nocturnal haemoglobinurea

618. Lithium SE all except?


ASIADH *

SABER revision for IM prometric exam| P a g e 60


BASICS
a- human behavior must be well understood
b- Information should be from cultural background
c- Doctors are only the health educators
d- Methods include pictures and videos (mass media)
620. A GIRL WITH RECURRENT e- Involve society members at early stage
ATTACKS OF ABD PAIN ANGIOEDEMA
627. Klinfelter syndrome:
,HER MOTHER ALSO HAD IT
Rare joint dislocation
Complement 1 deficiency MR
Airtic dissection
621. MEDIAN NERVE Supplies which Hypergonadotrophic hypogonadism*
muscles?
628. Remove cutaneous suture after:
Below external retinaculum!! (Flexor of course)
3-5 d
5-7 d
622. Patient with haemocystnemia
7-9 d
WHAT IS TRUE?
1-given folic acid decrease CVS risk 1.Tip of the nose innervated by branch:
2-decrease level of haemocystien does not affect the Ophthalmic
risk of CVS Trigeminal
3-given folic acid unmask pernicious anemia Maxillary

623. Regarding anatomy of CNS, WHAT 629. Most common site Langerhans cell
IS CORRECT histiocytosis
1-dual veins supply is mainly from external carotid Skin
vein Bone (skeleton)
2-opthalmic A is a branch of ant cerebral A Muscle
3-lacunar infarction could be pure sensory
5-Patient with Lt hemiparesis can turn his eye to the
630. Ophthalmic artery is a branch of:
right
Anterior cerebral artery*
Posterior cerebral artery
624. Fluid in proximal convoluted tubules
Middle cerebral artery
Isotonic Vertebral artery
Glucose not reabsorbed
631. Best treatment of frost bite?
Nerve block
625. 46 male pat CO from primary
Lidocaine + epinephrine
infertility semen show azospermia GA mimic…….. NPL
hormonal assay normal e anosmia D?
632. Median nerve
A -Klinfelter
B-Kallman Supplies thumb, index, middle finger
Supplies adductor pollicis
C-Turner

633. Pharmacokinetics
626. For health education programs to be
1st pass metabolism in liver
successful all are true except:
2.
SABER internal medicine prometric | P a g e 61
X-linked

634. Drug metabolism, one true?


Tetracycline absorption decrease with milk

635. Exposure to radiation not


penetrating tissue, only make burns?
Beta radiation*
Alpha radiation
Gamma radiation
XRays

636. One is true


Knee reflex L5
Ulnar nerve at wrist: loss of sensation of little finger
Radial nerve injury at middle ⅓ humerus >> wrist
drop*

637. A case of Ca 2.5 k 6 RF is


confirmed by
Urine analysis
Biopsy
Abd. U/S*
KFT, LFT

638. Which of the following is not


feminization?
Absent uterus
Female with pubic hair
XX karyotype

SABER revision for IM prometric exam| P a g e 62


routine pre-employment physical exam.
HEMATOLOGY He has always been healthy and his
examination is normal. Lab: HCT 35%
639. 60 year old male was refer to you
(38.8–46.4) , MCV: 63fL (79–93.3 fL)
after stabilization investigation show
, WBC: 6800/ml ,retics: 4000/ml
Hgb 18,5 g/dl , hect. 64% , RBC 7.8 ,
(0.7%) (0.8–2.3% red cells) Platelet:
WBC 15.3 & Plt. 570 Diagnosis:
27000/u1 his stool: -ve for occult blood
a- iron def. Anemia
b- Hgb pathy The most direct way to confirm
c- CLL suspected diagnosis:
d- 2ry polycythemia
A. Peripheral smear
e- Polycythemia rubra Vera
B. Measure Hb A2 level
Polycythemia vera is a blood disorder in which the C. G6PD screening
bone marrow makes too many red blood cells. D. Measure iron, TIBC and ferritin level
Polycythemia vera may also result in the E. Bone marrow stain for iron
overproduction of white blood cells and platelets.
Most of the health concerns associated with 642. 26-year-old man presented with
polycythemia vera are caused by a blood-thickening headache and fatigue. Investigations
effect that results from an overproduction of red
blood cells. revealed: Hb 8 g/dI , MCV 85 fL retics
10% ,All the following investigations are
640. 55 y/o female presented to ER
useful except:
because her family noticed skin A. Coomb’s test
discoloration. Has Hx of 5 kg loss over 3 B. Sickling test
C. Serum bilirubin
weeks. Her medical Hx is -ve apart D. Serum iron as it is normocytic anemia with
from vitiligo. Her examination is within reticulocytosis (hemolysis or hemorrhage)
E. Hb electrophoresis
normal except for scleral icterus & skin
jaundice. Her lx : WBC 2500 ,Plt 643. Increased bleeding time is seen in all
70000 (165–415 x 1000/mm3) , of the following except:
MCV 106, Hct 17, Retics count 15, a) Hemophilia
b) Scurvy
T.bil 3, which of the following test will
c) VwD (Von-Willebrand disease)
be more associated with the syndrome Bleeding
Condition PT PTT
she has: time
a- Chromosomal kariotype of bone marrow. Vitamin K
prolonged prolonged unaffected
b- Antiparietal cells antibodies. deficiency
c- Extrahepatic biliarv obstruction DIC prolonged prolonged prolonged
d- Decrease gastric fluid Haemophilia unaffected prolonged unaffected

641. 32 years old Saudi man from 644. Critical count of platelets which lead
Eastern province came to you for to spontaneous bleeding is:

SABER internal medicine prometric | P a g e 63


a) Kidneys
b) Liver
c) heart
d) skin
a) 1000 e) none of the above
b) 50.000
c) 75.000
d) 100.000 650. What should be the drug of choice in
e) 20.000 long term management of the disease?
(microscopic picture for sickle cells surrounded by
645. Serum ferritin reflects: neutrophils)
a) Total iron stores. a) Vit B12
b) Serum iron. b) Blood transfusion
c) Bone marrow iron. c) Hydroxyurea
d) None of the above. d) Steroids

646. Patient with fever, pallor petechiae, 651. Regarding sickle cell anemia, which
ecchymosis, CBC as WBC 2,800 Imm3 of the following statements are true?
a) Parvovirus cause aplastic crisis
,Hb 6 & plt 2900. next step of
b) 20% of the adults have splenomegaly
investigation: c) Hepatomegaly +splenomegaly result in acute
a) bone marrow aspiration hemolytic crisis in sicklers
d) Prognosis of sickle cell anemia is good
647. A 23 year old white female is
652. BF Showing sickle cell pt with crisis
diagnosed as having chronic ITP . Which
what Rx on long term
of the following will best predict a
BTx
favorable remission after splenectomy: Hydroxcarbamide
a. Presence of antiplatelet antibodies. Folic acid
b. Increased bone marrow megakaryocytes.
c. Absence of splenomegaly. 653. SICKLE CELL ANEMIA PATIENT IF
d. Platelet count of 170000/mm3 on corticosteroids.
e. Complement on platelet surfaces. MARRIED A NORMAL PERSON
POSSIBILITY OF SICKLE ANEMIA AND
648. Which of the following would most
TRAIT
likely indicate a hemolytic transfusion 0 and. 100
reaction in an anesthetized patent? 100 AND 0
25 AND 50
a) Shaking chills and muscle spasm
50 AND 50
b) Fever and oliguria
c) Hyperpyrexia and hypotention
d) Tachycardia and cynosis 654. CASE OF HUS
e) Bleeding and hypotension
655. YOUNG MALE FEVER WEIGHT
649. Which of the following organs is LOSS MALAISE 4 MO AGO , NOW
likely to receive a proportionately PALLOR HUGE SPLEEN , DIAGNSIS:
greater increase in blood flow? Leishmania

SABER revision for IM prometric exam| P a g e 64


CML 662. Protein C defeciency always come
Tropical splenomegaly
after 40 ys
656. Patient on Marivan INR 3 for gi
663. Anti-thrombin III associated with
endoscopy, what to do
Continue on low dose venous thrombosis most of presentation
Stop marivan Antithrombin III inherted by A D
Take heparin
664. Protein C defeciency is attributed to
657. Dvt in pregnant 32 weeks , what to factor V mutation and this may be the
give: cause of most of cases
Heparin
Warfarin
Aspirin 665. Picture of sickle cell anemia and
long scenario about frequent attacks
658. Paroxysmal nocturnal what is the drug that protecting from
hemoglobinuria: recurrent attacks
Only affects erythroid series 1-glucocorticoide
Gross hematuria 2-hydroxyuria
Needs warfarin to prevent thrombosis 3-folic acid
4-iron supply

659. MULTIPLE MYELOMA WHAT IS 666. What is the most common cancer

THE BEST PROGNOSTIC associated with hypercalcemia


1-thyroid
MEASUREMENT?
2-hodgikin
Calcium 3-MM
4-thymus
660. 55 ys old male has bronzed
skin,diabetous and abd pain with mild 667. Causes of renal impairment due to
tenderness,jaundice to complete the MM
diagnosis what is the next step : Hypercalcemia + Bence-Jones protein + myloma
infiltration
Paroxysmal nocturnal heamoglobinuria
1-s.iron saturation
Gross hematuria common present
2-liver biopsy
3-ERCP
4-PANCA 668. Sickle cell anemia
5-AMAb Parvovirus precipitate hemolytic crisis

661. regarding to hypercoagubale state 669. What is true about microangiopathic


all true except hemolytic anemia
Most common causes are inherted

SABER internal medicine prometric | P a g e 65


There is response to corticosteroids b-meagaloblastic aneamia
There is fragmented cells in blood film c-anemia of chronic disease
NB: This is my answer and i think it is right d-aplastic aneamia

670. A case with BM showing blue 673. Patient with DVT received warfarin,
stained cells (blasts) with few erythroid INR follow up, increased warfarin dose
red stained cells with no evidence of up to 15mg/day but INR is still 1.2
fibrosis and the patient has huge Incompliance
Warfarin resistance*
splenomegally with no hepatomegally

674. Prolonged B.T


Myelofibrosis
Chronic myeloid leukemia Decreased factor V
Myelodysplasia Decreased factor VII*
lymphoma Decreased factor IX
Decreased factor XI
671. Apatient wih huge splenomegally
675. Causes of microcytic anemia all
and CML, which of the following shoud
except:
be constant in treatment IDA
hydroxyuria Thalasemia
Interfeon alpha Scurvey
Predinsolone Sideroblastic anemia
Management
· hydroxyurea 676. Thalassemias
· interferon-alpha
Severe microcytic anemia from defective
· imatinib
· allogenic bone marrow transplant erythropiosis and decreased RBCs life
Imatinib
· inhibitor of the tyrosine kinase associated with the 677. Patient with tumor after
BCR-ABL defect chemotherapy = increased urea,
· very high response rate in chronic phase CML
increased cretinine
3.Regarding sickle cell anemia, which of D = tumor lysis syndrome

the following statements are true?


a) Parvovirus cause aplastic crisis
b) 20% of the adults have splenomegaly
c) Hepatomegaly +splenomegaly result in acute
hemolytic crisis in Sicklers
d) Prognosis of sickle cell anemia is good

672. patient e fatigue ,pallor e 4 week


fever CBC PLT 70000,WBC 3ooo,Hb 9
,ferttin 600 which least likely to be the
cause
a-iron deficiny aneamia
SABER revision for IM prometric exam| P a g e 66
NEUROLOGY
c- high blood pressure.
d- hyperlipidemia.
e- cigarette smoking.

678. Patient on haloperidol temp 40 C & 683. Pt come within 3 hrs C/O Lt side
rigidity weakness, examination revealed Lt side
NMS
hemiparesis, pulse 120/min irregular
Serotonin syndrome
with diastolic murmur at mitral area.
679. Alzehimer Dx 1st step of management:
Neurofibrillary tangle a) heparin
Hypometablism b) digoxin
CT e) EEG
d) carotid angiography
680. 20 years old male was brought to e) echo
the emergency room in very irritable
684. Patient suspected of having brain
condition, his pulse 100\min, temp
abscess, the most important q. in the
101 F, B.P 140\90, his condition
history is:
deteriorated and later on he developed
• frontal sinusitis
generalized tonic clonic fits. Which drugs • ear discharge.
will be best for his management: • head injury.
• bronchioctasis.
a) Fosphenytoin sodium
• Hx. of vomiting.
b) I.V diazepam
c) Phenobarbitone
d) Lorazepam 685. Which of the following statement is
false regarding Sydenham chorea?
681. 25-year-old student presented to a) Onset mostly in later age
your office complaining of sudden and b) It is involuntary movements
c) Involvement of face is also present
severe headache for 4 hours. History d) Involuntary movements disappear during sleep
revealed mild headache attacks during
686. A young man developed brief period
the last few days. On examination:
of consciousness without falling to the
agitated and restless. What Dx must be
ground. What is the diagnosis?
considered this case?
a) Simple partial seizures
A. Severe migraine attack
b) Absent seizure
B. Cluster headache
c) Complex partial seizure
C. Subarachnoid hemorrhage
d) Myoclonic seizure
D. Hypertensive encephalopathy
E. encephalitis
687. Patient woke up with Rt side
682. Greatest risk for stroke: hemiparesis after 6 hour sleep what to
a- DM. do?
b- family history of stroke.
Streptokinase

SABER internal medicine prometric | P a g e 67


Alteplase 1-aspirin
Heparin 2-halidol
Oral or rectal aspirin after CT exclude hge 3-diazepam
4-B blocker
688. TIA for 2hour in 50 yr old man
694. Young male with weakness started
what to do
Contrast enhanced CT brain in his legs:
Non -Contrast enhanced CT brain Guillan baree
MRA
MRI
695. trigeminal neuralgia WHAT IS THE
689. what cause absent ankle &upgoing
WRONG
plantar 1-causing lanciting pain
B6 deficiency 2-excerbating by cold weather
Tabes dorsalis 3-bec of side effect of carbamezipine now limit its use
Motor neuron disease 4-can cause weight loss
Conusmedullaris lesion Recurrent TIAs

690. Young male had Jerky hand 696. A case of CV stroke management
movement ,3 generalized tonic clonic with history of peptic ulcer ,
fits , what to give a) ASA
b) Clopidogrel
Clonazepam
haloperidol
697. A picture of brain CT for a patient
691. Leg spasticity , diplopia, history of with hemiplegia since 90 min and CT
lost vision recovered: shows ischemia, the treatment is
Multiple sclerosis Aspirin
Recurrent TIAs TPA
Guillan Baree LMWH
warfarin

692. Vertigo,diplopia,dysarthria, face 698. About Alzhiemer disease


numbness on right, lost sensation on left Gradual decline in the cognitive function

. What artery affected


699. The best Drug for prophylaxis
Vertebral
Branch of basilar against migraine is
Pst cerebral
Inf cerebellar Amyltriptaline
Sumatriptan
693. After attending a party with her
700. A case with a picture showing that
friend she became agitated
the tongue of the patient has whitish
,feverish,palpitation and in ER she has
patches (mostly that of candidiasis) and
fits WHAT IS YOUR MANGMENT

SABER revision for IM prometric exam| P a g e 68


says that the paitient is having HIV Bilateral arterial angiography
CT brain
infection and the CDC is 350, SO the Plain X-ray skull
CDC classification of the patient is
A1 705. Dytharthria, ataxia, dizziness,
B2 diplopia, most probably diagnosis?
C1
Carotid artery insufficiency
C3
Basilar artery insufficiency*
Vertebral artery insufficiency
701. Patient e RT side hemiparesis’ Main stem lesion
mostly due to
a-cerebral embolisem 706. Fredrich’s ataxia all ataxia
b-Preiphral embolization Caused by degradation of dorsal root ganglia
Autosomal dominant*
702. TIA for 2hour in 50 yr old man Loss of deep tendon reflexes

what to do
707. 22 ys old patient with clonic
Contrast inhanced CT brain
Non -Contrast inhanced CT brain seizures (MRI) picture
MRA brain abscess
MRI
708. MS all except:
703. Lateral medullary syndrome culprit Loss pupillary reaction
territory Increased oligoclonal bands* ??
Gradual onset in 20% of patients
PICA
Female to male ratio is 2:1
MCA
Vertebra
Basilar 709. Rt sided facial pain, numbeness,
painful mastication, treatment?
Lateral medullary syndrome Phenytoin
also known as Wallenberg's syndrome, occurs Carbamazebine*
following occlusion Valproaic acid
of the posterior inferior cerebellar artery Diazepam
Cerebellar features
· ataxia
710. EEG is helpful in investigating all
· nystagmus
except?
Brainstem features Sleep disorders
· ipsilateral: dysphagia, facial numbness, cranial nerve Epilepsy
palsy e.g. Horner's Encephalitis
· contralateral: limb sensory loss Brain tumor*
Hepatic coma
704. Patient presented to ER by loss of
consciousness, hge at eye, eye movement 711. Neuroleptic malignant syndrome all

intact, the best initial investigation? except:


EEG It does occurs with small dose of offending drug *

SABER internal medicine prometric | P a g e 69


712. Parkinsonism all are true except
Intention tremors*
Rigidity
Micrographia
Cog-wheel spasticity

713. Muscle fasiculations occur in


LMNL

‫المذكرة دي أولية لحين االنتهاء من النسخة النهائية‬


‫ شهور إن شاء هللا‬4 ‫فى خالل‬

‫وأخر دعوانا أن الحمد هلل رب العالمين‬

SABER revision for IM prometric exam| P a g e 70

You might also like